You are on page 1of 21

Jak zabra si za potn potg i znale Grup Trzymajc Rozwizanie

Zastosowanie pojcia rzdu i generatora w rozwizywaniu zada olimpijskich


Autor: Tomasz Peka
* * *
1 Wstp
Niniejsza praca ma na celu przedstawienie podstawowych poj Teorii Grup i ich zastosowa w rozwizywaniu
zada z Olimpiad Matematycznych. Ta stosunkowo prosta teoria przede wszystkim daje moliwo intuicyjnego
podchodzenia do pewnego typu zada (gwnie z Teorii Liczb) oraz jest przydatnym narzdziem redukujcym ilo
rozpatrywanych przypadkw. Oczywicie niemal kade przedstawione poniej zadanie da si rozwiza metodami
zawartymi w licealnych podrcznikach (i takie rozwizania zwykle s podawane jako wzorcowe), niemniej jednak
dziki zastosowaniu metod tu przedstawionych wiele wcale nietrywialnych zada staje si praktycznie natychmiastowymi.
1.1 Podstawowe pojcia
1
1. Rzd
Mwimy, e t jest rzdem a mod n, (gdzie (n, a) = 1), jeeli t jest najmniejsz liczb naturaln tak, e a
t
1
(mod p). Zapisujemy to t = ord
n
a. Z twierdzenia Eulera wynika, e taka liczba zawsze istnieje.
Podstawowe wasnoci rzdu:
1. Liczby a
0
, a
1
, . . . , a
t1
s mod p wszystkie rne. Istotnie, gdyby dla pewnych k i l byo a
k
a
l
, to wtedy
a
]kl]
1, co jest niemoliwe, bo [k l[ < t.
2. Dla kadego k takiego e a
k
1 (mod p) mamy t[k (w szczeglnoci zawsze t[(p)). Rzeczywicie, niech
k = tq +r przy t > r 0, wtedy 1 a
k
(a
q
)
t
a
r
a
r
(mod p) i std ju mamy, e r = 0.
2. Generator
Jeeli (g, n) = 1 i ord
n
g = (n), to g nazywamy generatorem mod n. Nazw t uzasadnia fakt, e podnoszc g
do kolejnych potg mod n moemy wygenerowa wszystkie liczby mniejsze od n i wzgldnie pierwsze z n (patrz 1.
wasno rzdu).
Liczba n ma generator wtedy i tylko wtedy, gdy jest postaci 2, 4, p

, 2p

, gdzie Z a p P 2
2
. Dowd tego
twierdzenia jest rozbudowany i znajduje si w Dodaku A, tutaj przytocz jedynie przydatne skadajce si na
lematy:
1. Dla kadej liczby nieparzystej a i cakowitej przy > 3 zachodzi 2

[a
2
2
1.
2.

d]n
(d) = n
3. Jest (t) liczb 0 < a < p takich, e ord
p
a = t
4. Jeli g jest generatorem mod p
2
, to jest generatorem mod p

3. Waluacja
Mwimy, e k jest waluacj p-adyczn n (zapisujemy v
p
(n) = k), jeli p
k
[[n (p
k
dzieli dokadnie n), czyli p
k
[n i
p
k+1
n. Oczywicie v
p
(xy) = v
p
(x) +v
p
(y) i jeli y[x to v
p
(
x
y
) = v
p
(x) v
p
(y).
Niech p P, a, b Z, n Z
+
, p a, p b. Zachodzi nastpujcy lemat o waluacjach
3
:
a) jeli p ,= 2 i p[a b, to v
p
(a
n
b
n
) = v
p
(a b) +v
p
(n),
b) jeli p = 2 i p[
a
2
b
2
2
, to v
p
(a
n
b
n
) = v
p
(
a
2
b
2
2
) +v
p
(n).
Dowd:
a) Niech n = p

k i p k. Bdziemy rozumowa indukcyjnie ze wzgldu na przy ustalonym k.


Poniewa z zaoenia a b (mod p), mamy a
i
b
i
(mod p) dla kadego i. Std
a
n
b
n
ab
=

n1
i=0
a
i
b
n1i

n1
i=0
b
n1
nb
n1
, 0 (mod p) bo p b i p n z zaoenia. Teza zachodzi wic dla = 0.
Zamy, e zachodzi dla pewnego 0, czyli p

[[
a
n
b
n
ab
. Aby dowie, e teza zachodzi te dla +1, wystarczy
1
a do ostatniego rozdziau nie bd powoywa si na Teori Grup sensu stricto, aby nie zaciemnia wywodu
2
Od tej pory, jeeli nie zostanie zaznaczone inaczej, litera p bdzie zawsze reprezentowa liczb pierwsz nieparzyst
3
ang. lifting the exponent lemma
1
(wszak k ustalilimy) pokaza, e p
2

a
np
b
np
a
n
b
n
.
Poniewa p[ab, mamy a = xp+b, a wic a
i
= (xp+b)
i
ib
i1
xp+b
i
(mod p
2
) (pozostae czynniki rozwinicia
s podzielne przez p
2
). Czyli teraz:
a
np
b
np
a
n
b
n
=
p1

i=0
a
ni
b
n(p1i)

p1

i=0
(ixpb
n(i1)
+b
in
)b
n(p1i)

p1

i=0
(ixpb
n(p2)
+b
n(p1)
) pa
n(p1)
(mod p
2
),
skd wprost wynika teza.
b) Znw, niech n = 2

k przy 2 k i rozumujemy indukcyjnie.


Udowodnimy, e teza zachodzi dla = 1. z 2[
a
2
b
2
2
otrzymujemy 2[a b. W takim razie a b (mod 2) i
a
2k
b
2k
a
2
b
2
=

k1
i=0
a
2i
b
2k2i2


k1
i=0
a
2k2
ka
2k2
1 (mod 2), co koczy dowd dla = 1. Zamy, e
2
1
[[
a
n
b
n
a
2
b
2
. Mamy a
n
1 b
n
(mod 4) z parzystoci n, wic a
n
+ b
n
2 (mod 4), czyli 2[[a
n
+ b
n
, a wic
2

[[
a
2n
b
2n
a
2
b
2
, co naleao dowie.
4. Odwrotno modulo
Jeeli (a, n) = 1 to istnieje taka liczba b, e ab 1 (mod n). Tak liczb oznacza bed w niniejszej pracy jako
1
a
,
jeeli wic w kongruencji pojawi si nagle np.
2
5
, oznacza to bdzie dwukrotno odwrotnoci liczby 5, oraz e
na pewno 5 jest wzgldnie pierwsza z moduem.
2 Prawie jak logarytmy
Oczywicie, jeli ord
n
a[d i (mod d), to a

(mod n) (dla d = ord


n
a otrzymujemy rwnowano). W
szczeglnoci powysza implikacja zachodzi dla d = (n) (twierdzenie Eulera). Ta z pozoru trywialna obserwacja
ju jest mocnym narzdziem przy rozpatrywaniu potg o tych samych smych podstawach. Wemy zadanie:
Niech n, a Z
+
, n > 1. Niech a
1
= a oraz a
k+1
= a
a
k
dla k 1. Dowie, e cig (a
n
mod n) jest od pewnego
miejsca stay.
Rozwizanie:
Niech max(a
k
, n), k Z
+
= d. Wtedy od pewnego miejsca cig jest mod d stale rwny zero. Wystarczy wic
udowodni tez dla (n, a) = 1.
Pokaemy wicej - e cig ten ustala si najpniej po n-tym wyrazie. Dla n = 2 teza jest oczywista (albo
wszystkie wyrazy s parzyste, albo nieparzyste). Zamy, e teza zachodzi dla wszysstkich liczb mniejszych od n.
W szczeglnoci zachodzi dla (n). Mamy wic c a
(n)
a
(n)
. . . (mod (n)). Korzystajc ze wspomnianej
wasnoci wnioskujemy, e w takim razie c
a
a
a
(n)
. . . (mod n), a wic cig ustala si po (n) + 1-tym
wyrazie.
Wasno t stosuje si jednak najczciej podstawiajac za a generator mod n, wtedy d = (n). Wane, e w
tej sytuacji mamy rwnowano, a to w poczeniu z faktem, e dla n posiadajcych generator (n) jest do duze,
eliminuje pokan liczb maych przypadkw. Wemy na przykad zadanie:
Rozwiza w cakowitych dodatnich x, y, z rwnanie 5
x
3
y
= z
2
Rozwizanie:
Biorc dane rwnanie mod 3 uzyskujemy, e x = 2k, a wic istniej takie liczby cakowite , e + = y i
3

= 5
k
z, 3

= 5
k
+z. Jeliby > 0, byoby 3[3

+ 3

= 2 5
k
, sprzeczno, a wic = 0, czyli 1 + 3

= 2 5
k
.
3 jest generatorem mod 5
k
, czyli z powyszej rwnoci wynika, e
(5
k
)
2
(mod (5
k
)), z czego dostajemy, e

(5
k
)
2
= 25
k1
. Mamy wic: 25
k
= 1+3

> 3
25
k1
, co jest moliwe tylko gdy k = 1. A wtedy y = = 2 i z = 4.
Intuicyjnie, rzecz ma si niemal tak jak ze szkolnymi logarytmami. Uwaa jednak naley, aby zawsze (n, a) = 1
(w przeciwnym wypadku w ogle nie mona mwi o rzdach!), a take na to, e w tej wasnoci rwnowano
zachodzi tylko wtedy, gdy wykadniki rozpatrujemy modulo ord
n
a.
Zadania:
1. Rozwiza w cakowitych x, y, z rwnanie 5
x
+ 7
y
= z
3
2. ** Znale wszystkie takie wielomiany P o wspczynnikach cakowitych, e dla kadej liczby pierwszej p
zachodzi P(p)[2
p
2
2
3 Bijekcja z kapelusza
Zamy, ze w pewnym zadaniu mamy cig, ktry jest permutacj Z

p
. Jeli jednak g jest generatorem mod p, to na
pewno (1. wasnos rzdu) g
0
, g
1
, . . . , g
p2
= Z

p
(liczby wite mod p). Okazuje si, e duo atwiej jest dowodzi
tezy, gdy wemiemy za dany zbir - zbir zdeniowany powyej.
Jako przykad udowodnimy bardzo wany lemat:
Jeli p 1 k, to p[1
k
+ 2
k
+ + (p 1)
k
Rozwizanie:
Wemy wic zamiast cigu (1, 2, . . . , p 1) cig (g
0
, . . . , g
p2
). Wtedy

p1
i=1
i
k


p2
j=0
(g
j
)
k


p2
j=0
(g
k
)
j

(g
k
)
p1
1
(g
k
)1
(mod p). Mozemy tak zapisa, gdy z zaoenia k p 1 = ord
p
g, a wic mianownik jest rny od zera.
Licznik za, z MTF, jest rwny zeru (mod p), a to jest teza.
Zastosowania tego lematu zostan omwione w nastpnym paragrae.
Oczywicie, eby stosowa ten trik, nie musimy mie danego caego ciagu Z

p
. Bardzo czsto rozumowanie staje
si atwiejsze, gdy za dan liczb a tak e (a, p) = 1 podstawimy g

dla g-generatora mod p i pewnej liczby ,


wzitej mod p 1. Rozwamy nastpujce zadanie:
Udowodni, ze jeli n Z
+
i n[3
n
+ 4
n
, to 7[n
Rozwizanie:
Niech p bdzie najmniejszym dzielnikiem pierwszym n (wida, e p ,= 2). Dla pewnych x ,= y jest g
x
3 (mod p),
g
y
4 (mod p) i z danej podzielnoci wynika, e g
xn
+g
yn
0 (mod p), co po przeksztaceniach daje g
2n(xy)
1
(mod p) i z wasnoci rzdu mamy, e p1[2n(xy). Gdyby byo (p1, n) > 1, wiedy n miaoby dzielnik pierwszy
mniejszy od p, niezgodnie z zaoeniem. Jest wic (p 1, n) = 1, i z naszej podzielnoci wynika, e
p1
2
[x y.
Ale x, y [0, p 1], wic [x y[ =
p1
2
. Niech x > y (rozumowanie w drugim przypadku jest analogiczne). Po
przeksztaceniach otrzymujemy, e x =
p1
2
+y, a wic g
p1
2
+y
3 (mod p) i g
y
4 (mod p). Poniewa (p, 4) = 1
moemy podzieli te kongruencje stronami i podnie do kwadratu, uczyniwszy to otrzymamy g
p1

9
16
(mod p),
a e oczywicie g
p1
1 (mod p), to po przeksztaceniach otrzymamy 0 7 (mod p), skd wynika teza.
Na koniec pokaemy jeszcze jeden fakt z dowodem bardzo podbnym do poprzedniego:
Ile rozwiza w (x, y) Z

p
2
ma kongruencja x y
n
(mod p) (przy n[p 1)?
Rozwizanie:
Chcemy sprawdzi, kiedy a
n
b
n
(mod p). Znw podstawiamy za a g
x
, za za b - g
y
i zamy x > y. Nasze pytanie
sprowadza si do sprawdzenia, kiedy g
nx
g
ny
(mod p), czyli g
n(xy)
1 (mod p). Ale skoro rzd ord
p
g = p 1,
to z wasnoci rzdu p 1[n(x y), a wic
p1
n
[(x y). Wida teraz, e n-te potgi liczb g
0
, g
1
, ..., g
p1
n
1
bd
dawa rne reszty mod p, i e bd to wszystkie takie reszty (Bo
_
g
k
p1
n
+r
_
n

_
g
k
p1
n
_
n
g
rn
g
rn
z MTF,
a wic kada potga o wykadniku wikszym od
p1
n
ma swj odpowiednik we wspomnianym wyej cigu). Nasze
rwnanie ma wic dokadnie
p1
n
rozwiza.
Rozwizania te, analogicznie do pojcia reszty kwadratowe, nazywa bdziemy resztami n-potgowymi mod p.
atwo zauway, ze w oglnym przypadku jest ich
n
=
p1
(n,p1)
. Np. przy rozwizywaniu rwna diofantycznych
zaley nam, by
n
byo jak najmniejsze - popatrzmy na rwnanie x
6
= y
5
+ 24
Rozwizanie: Bierzemy je mod 31, bo 31 P i 31 1 = 30 = 6 5. Szybko
4
dostajemy, e reszty 5-potgowe mod
31 to 1, 26, 25 i 1, 26, 25 (bo 2 5), czyli 1, 5, 6, 25, 26, 30. Z kolei reszt 6-potgowych jest tylko 5: 1, 2, 4, 9, 16.
Chobymy chcieli, 24 z tego nie uzyskamy, std wniosek, ze rwnanie nie ma rozwiza.
Jak si okazuje, wikszo rwna diofantycznych, take olimpijskich, jest wanie w taki sposb ukadana.
Powysze metody sprawdzaj si, jeli wiemy, e p jest pierwsze. Jeeli jednak nie mamy tej informacji, ale mimo
to co o p wiemy, przydatne staje si Twierdzenie Dirichleta. Przypomnijmy, mwi ono, e w dowolnym postpie
arytmetycznym postaci an + b, jeli tylko (a, b) = 1, istnieje nieskoczenie wiele liczb pierwszych (jego dowd
znacznie wykracza poza ramy niniejszej pracy). Jeeli w zadaniu pojawia si dodatkowo sformuowanie udowodni,
e istnieje nieskoczenie wiele p, prawie na pewno chodzi o zastosowanie tego twierdzenia. Rozpatrzmy przykad:
Pokaza, e istnieje nieskoczenie wiele n Z
+
takich, e najwikszy dzielnik pierwszy liczby n
4
+ 1 jest wikszy
ni 2n
Rozwizanie:
Z Twierdzenia Dirichleta wiemy, e istnieje nieskocznie wiele liczb pierwszych p takich, e 8[p 1. (rozwaamy
postp arytmetyczny 8n+1, (8, 1) = 1). Niech n bdzie rozwizaniem kongruencji x
4
+1 0 (mod p), co rwnoway
dwu warunkom: ord
p
n[8 i ord
p
n 4, czyli ord
p
n = 8. Takich liczb jest (wedle jednego z lematw dowodzonych w
Dodatku A) dokadnie (8) = 4, mianowicie b, b
3
, b
5
, b
7
, gdzie b = g
p1
8
a g jest generatorem mod p. Poniewa
b b
5
(mod p), jedna z tych liczb jest wiksza od
p
2
, a druga mniejsza. Wemy za n t mniejsz. Wtedy p[n
4
+ 1
i n <
p
2
, co chcielimy pokaza.
4
Po prostu sprawdzamy kilka pierwszych reszt, dopisujemy im minusy i argumentujemy, ze wiecej by nie moe
3
Zadania
1. Wykaza, e istnieje taka permutacja (x
1
, x
2
, . . . , x
p1
) zbioru 1, 2, . . . , p 1, e liczby
x
1
, x
1
x
2
, x
1
x
2
x
3
, . . . , x
1
x
2
...x
p1
s rne mod p.
2. a
1
, a
2
, . . . , a
p1
= b
1
, b
2
, . . . , b
p1
= Z

p
. Pokaza, e istniej takie liczby k ,= l, e p[a
k
b
k
a
l
b
l
3. Dla liczby pierwszej p, naturalnej k i naturalnej t takiej, e (t 1, p 1) = 1 deniujemy zbir S nastpujco:
[S[ = k,
S 1, 2, . . . , p 1
Iloczyn mod p dowolnych t (niekoniecznie rnych) elementw S naley do S.
a) Pokaza, e iloczyn mod p dowolnych dwch elementw S naley do S.
b) Wyznaczy liczb takich S w zalenoci od k przy ustalonym p
4. * Po lewej stronie tablicy napisano liczby 1, 2, 3, . . . , p1 za po prawej stronie liczb 0. Wykonujemy cig p1
ruchw, z ktrych kady przebiega nastepujaco: Wybieramy jedn z liczb napisanych po lewej stronie tablicy,
dodajemy j do wszystkich pozostaych liczb na tablicy, po czym wymazujemy wybran liczb. Rozstrzygn,
dla jakich wartoci p mona w kolejnych ruchach wybiera liczby w taki sposb, by liczba pozostaa na tablicy
po wykonaniu wszystkich ruchw bya podzielna przez p.
4 Jeszcze o magicznych bijekcjach
W poprzednim paragrae udowodnilimy, e jeli p 1 k, to s
k
def
= 1
k
+ + (p 1)
k
0 (mod p). Ten lemat
znajduje zastosowanie w zadaniach wykorzystujcych wielomiany p1-szego stopnia mod p. Niech P Z[X] bdzie
takim wielomianem. Mamy:
p1

i=1
P(i)
p1

i=1
p1

k=0
a
k
k
i

p1

k=0
a
k
p1

i=1
k
i

p1

k=0
a
k
s
k
a
p1
s
p1
a
p1
.
Rozwamy nastpujcy problem:
Znale maksymalny stopie wielomianu P o wspczynnikach ze zbioru 0, 1, 2, . . . , p 1 stopnia mniejszego ni
p, takiego, e dla dowolnych m, n Z jeeli p[P(n) P(m) to p[n m.
Rozwizanie:
Jeli rozwaymy P jako funkcj Z
p
Z
p
, wystarczy pokaza, e P jest bijekcj.
Jako e x
p2
dla (x, p) = 1 jednoznacznie wyznacza x mod p (multiplikatywna odwrotno), a gdy p[x zawsze wynosi
0 mod p, P(x) = x
p2
spenia warunki zadania. Udowodnimy, e aden wielomian stopnia p 1 ich nie spenia.
Zamy przeciwnie, e istnieje bijekcja P(x) =

p1
k=0
a
k
x
k
. Wtedy w szczeglnoci:
0
p

i=1
i
p

i=1
P(i)
p

i=1
p1

k=0
a
k
i
k

p1

k=0
a
k
p

i=1
i
k

p1

k=0
a
k
s
k
a
p1
(mod p),
a wic 0 a
p1
, sprzeczno z denicj wielomianu.
O triku z zamian kolejnoci sumowa warto pamita zawsze, gdy wiemy, e sumy jakiej konkretnej postaci si
zeruj - tak za informacje czsto daje nam rozumowanie indukcyjne. I znw posuymy si przykadem:
Niech 1 k p 1 , k Z. Udowodni, e
p[1 2 ... k + 2 3 ... (k + 1) + + (p k) (p k + 1) ...(p 1),
(powysza suma zawiera
_
p1
k
_
skadnikw).
Rozwizanie:
Dla k = 1 mamy po prostu p[

p1
i=1
i =
1
2
p(p 1), co oczywicie jest prawd, dla k = 2 mamy p[

1i<p
=
1
2
((

p1
i=1
i)
2

p1
i=1
i
2
), co po zastosowaniu odpowiednich wzorw rwnie daje zgodno. Zamy, ze teza zachodzi
dla wszystkich liczb mniejszych od k i niech F
i
= A Z

p
: [A[ = i. Chcemy dowie, e p[

AF
k

aA
a.
Dodatkowo, wiemy, e dla kadego A Z

p
zachodzi

aZ

p
\A
=

p1
i=1
i

aA
a

aA
a (mod p). Majc to
na uwadze, zapiszmy:

AF
k

aA
a =
1
p k + 1

AF
k1

aA
a

bZ

p
\A
b
4
poniewa 0 < p k + 1 < p, wystarczy, e dowiedziemy:
p [

AF
k1

aA
a

aA
a =

aZ

p
a

AF
k1
,Aa]

bA
b =

aZ

p
a
2

AF
k2

bA
b,
ale p[

AF
k2

bA
b z zaoenia indukcyjnego.
Lemat p[1
k
+ +(p 1)
k
zosta udowodniony w oparciu o generator. Jak nietrudno si domyli, jeeli wemiemy
liczb a ,= 1 tak, e ord
p
a = d, zachodzi bdzie
d1

i=0
(a
i
)
k
=
d1

i=0
(a
k
)
i
=
(a
d
)
k
1
a
k
1
0 (mod p).
* * *
Zobaczymy teraz, jak zdobyte dotychczas umiejtnoci przydadz si w rozwizaniu zadania p.t. nietrywialne
sumy z AMM:
Niech p, q P, r Z
+
, q[p1 i q r. Niech ponadto a
1
, a
2
, . . . , a
r
bd takimi liczbami cakowitymi, e

r
i=1
a
i
0
(mod p). Pokaza, e co najmniej jedna z tych liczb jest podzielna przez p.
Rozwizanie:
Zamy przeciwnie, e wszystkie a
i
s wzgldnie pierwsze z p. Niech g bdzie generatorem mod p i a
i
g
t
i
(mod p). Niech a = g
p1
q
. Oczywicie wtedy ord
p
a = q oraz a
p1
q
i
a
t
i
. Rozpatrzmy wielomian f(x) =

r
i=1
x
t
i
.
Mamy f(a) 0 i f(1) = r.
Niech teraz g(x) =

q1
i=1
f(x
j
). Poniewa dla kadego j [1, q 1] istnieje takie i [1, q 1], e ij 1 (mod q),
dla i-tej z liczb a, a
2
, . . . a
q1
, j-ty z czynnikw ww. iloczynu przystaje do f(a) 0 (mod p) (wystarczy bowiem, by
wykadnik przystawa do 1 mod ord
p
a = q z wasnoci rzdu). Stad wniosek, ze liczby a, a
2
, . . . , a
q1
s pierwiastami
g. Ponadto, g(1) =

q1
i=1
f(1) = r
q1
.
Powysze wasnoci g spenia rwnie wielomian h powstay z wzicia wszystkich wykadnikw mod q (patrz paragraf
1). Poniewa h ma q 1 pierwiastkw, musi by stopnia co najmniej q 1, jednake z jego konstrukcji wynika, e
jest on stopnia co najwyej q 1, a std deg h = q 1. Niech teraz h(x) =

q1
i=0
c
i
x
i
. Rozpatrzmy wielomian:
v(x) = 1 +x +x
2
+ +x
q1
.
v i h maj te same pierwiastki i s tego samego stopnia, wic s rwne z dokadnoci do staej (wszystko mod p).
Std wniosek, e dla kadego i [0, q 1]c
i
c (mod p) dla pewnego c. Jednake r
q1
= h(1) =

q1
i=0
c
i
q c
(mod p). Poniewa z warunkw zadania r
q1
< p, a c byo dobrane mod p, mamy r
q1
= cq, a std q[r, sprzeczno
z warunkami zadania.
Zadania
1. Wyznaczy wszystkie wielomiany p1-szego stopnia P takie, e p[P(x)P(y) P(xy) dla dowolnych x, y Z

p
.
2. Niech a, b, c Z, p P oraz p a i p b
2
4ac. Pokaza, e
p

k=1
_
ak
2
+bk +c
p
_
=
_
a
p
_
,
gdzie
_
u
v
_
oznacza symbol Legendrea.
5 Dont underestimate the power of the right order
W poprzednich zadaniach bralimy wykadniki mod (p) bez zastanowienia: teraz bedziemy podchodzi do wykadnikw
bardziej metodycznie i ostronie. Nie zawsze bowiem mamy ten komfort, e wszystkie liczby s wzgldnie pierwsze
z moduem lub e istnieje generator: przykadem s zadania typu wyznaczy wszystkie n Z
+
, e zachodzi
podzielno - takimi bdziemy si w niniejszym paragrae zajmowa. Okazuje si jednak, e umiejtne dobranie
liczby, ktra ma by rzdem nierzadko od razu na mocy wasnoci rzdu (ord
n
a[(n)) doprowadza do sprzecznoci.
Na przykad jeli mamy podzielno n[a
p
1, wyznaczenie rzdu a sprowadza si do rozpatrzenia dwch przypadkw.
Rozpatrzmy zadanie:
Udowodni, e jeli p jest liczb pierwsz, to liczba p
p
1 posiada dzielnik pierwszy, ktry daje reszt 1 z dzielenia
przez p
Rozwizanie:
Niech q bdzie dzielnikiem pierwszym liczby p
p
1. Mamy p
p
1 (mod q) i oczywicie (p, q) = 1. Niech t = ord
q
p.
Z wasnoci rzdu mamy, e t[p, czyli t = 1 lub t = p. Jeli t = p, to z MTF p[q 1 i mamy tez. Jeeli za t = 1,
czyli p 1 (mod q), dla kadego q, wtedy
p
p
1
(p1)
= (

p1
k=0
p
k
) p 1 (mod q), co oznacza, e suma ta nie jest
podzielna przez swj dzielnik pierwszy, sprzeczno.
Istniej dwie metody podchodzenia do bardziej oglnych zada tego typu, przy czym zazwyczaj trzeb zastosowa
obie naraz:
5
5.1 Metoda nieskoczonego schodzenia i najmniejszy dzielnik pierwszy
Oglna zasada metody nieskoczonego schodzenia polega na tym, e bierzemy najmniejszy element zbioru i udowadniamy,
e istnieje jeszcze mniejszy, a stad wniosek, ze zbir by niepoprawnie zdeniowany. Wane jest przy tym, by w tym
zbiorze istnia element najmniejszy: dobrymi przykadami takich zbiorw s podzbiory Z
+
(ale Z ju nie), na
szczcie takimi tylko bdziemy si zajmowa.
Jeeli na oko wida, e rozwizaniem naszego zadania jest 1 lub maa liczba pierwsza, dobrm zbiorem do rozwaania
jest zbir dzielnikw pierwszych tej liczby. Wemy prosty przykad:
Dowie, e jeli n Z
+
i n[2
n
1, to n = 1.
Rozwizanie:
Dla n = 1 teza zachodzi. Zamy, e n > 1. Poniewa 2
n
1 jest nieparzyste, n musi by nieparzyste. Niech p ,= 2
bdzie najmniejszym dzielnikiem pierwszym n. Z warunkw zadania wynika 2
n
1 (mod p) Niech t = ord
p
2. Z
wasnoci rzdu mamy t[n i t[p 1, a std t < p. Jeeliby t > 1, n posiadaoby dzielnik pierwszy mniejszy od p,
sprzeczno. Wic t = 1, czyli 2 1 (mod p), a std p = 1, sprzeczno.
Oczywicie, nie zawsze schodzimy po dzielnikach pierwszych. Czsto warto najpierw wzi dowolny dzielnik, a
potem dopiero zauway, co bdzie najmniejsze:
Znale wszystkie n Z
+
takie, e n[2
n1
+ 1
Rozwizanie:
Dla n = 1 teza zachodzi, zamy, e n > 1 i niech p bdzie dowolnym dzielnikiem pierwszym n (oczywicie p ,= 2).
Mamy wtedy 2
n1
1 (mod p), a std 2
2(n1)
1 (mod p). Niech t = ord
p
2. Mamy t[2(n 1) i t n 1.
Niech teraz n 1 = 2
k
u przy u nieparzystym. Wida, e musi by 2
k+1
[t (gdyby 2 dzielio t w mniejszej potdze,
byoby t[n1). Z wasnoci rzdu mamy z kolei, e t[p1, a wic i 2
k+1
[p1, czyli p 1 (mod 2
k+1
), i zachodzi to
dla kadego dzielnika pierwszego n. Std wniosek, e n 1 (mod 2
k+1
), a wic 2
k+1
[n 1 = 2
k
u, sprzeczno.
5.2 Dowolny dzielnik pierwszy
Jeeli wydaje si, e rozwizanie bdzie bardziej skomplikowane, zamiast bra najmniejszy dzielnik pierwszy, moemy
stara si znale jak wspln cech wszystkich dzielnikw pierwszych, biorc dowolny z nich. Wemy rwnie prosty
przykad:
Dowie, e kady dzielnik liczby 2
2
n
+ 1 (n-ta liczba Fermata) jest postaci 2
n+1
k + 1 dla pewnego k Z.
Rozwizanie:
Iloczyn liczb postaci 2
n+1
k+1 rwnie jest postaci 2
n+1
k+1, wic moemy ograniczy si do dzielnikw pierwszych.
Niech p bdzie dzielnikiem pierwszym 2
2
n
+ 1. Mamy 2
2
n
1 (mod p), co po podniesieniu do kwadratu daje
2
2
n+1
1 (mod p). Niech t = ord
p
n. Wida, e t 2
n
i t[2
n+1
, czyli t = 2
n+1
Z drugiej strony, 2
n+1
= t[p 1, a
wic p 1 = 2
n+1
k dla pewnego k, skd wprost wynika teza.
5.3 Generator kontratakuje
Metody opisane powyej dziaaj, jeli dowolna liczba naturalna, dla ktrej ma zachodzi jaki warunek, jest
wykadnikiem. Jeli za mamy dan liczb pierwsz, a mamy dowie jak wasno pewnej podstawy potgi, warto
za ni podstawi generator - wtedy wasno rzdu daje nam mocne grne ograniczenie na wykadnik. Popatrzmy
na przykad:
Wyznaczy wszystkie liczby pierwsze p, q takie, e 3pq[a
3pq
a dla kadego a cakowitego.
Rozwizanie:
Gdyby p = 3, dla a = 2 mielibymy w szczeglnoci (2
9
)
p
2 (mod 9), Czyli 1 2 (mod 9), sprzeczno. Rwnie
atwo sprawdzamy, e p ,= 2 (wystarczy podstawi za a := 5). Mamy wic p, q ,= 2, 3 Wemy za a generator mod p.
Udowodnimy najpierw, e p ,= q. Istotnie, gdyby byo p = q, to a
3p
2
1
1 (mod p), z wasnoci rzdu mielibymy
wtedy p 1[3p
2
1 = (p 1)(p + 1) + 2p
2
, a wic p 1[2p
2
, a std p = 3, sprzeczno.
Z MTF 0 a
3pq
a a
3q
a (mod p), skd mamy a
3q1
1 (mod p). Z wasnoci rzdu, w tym wypadku rwnego
p 1, otrzymujemy, e p 1[3q 1. Przeprowadzajc analogiczne rozumowanie dla q otrzymujemy, e q 1[3p 1.
Zamy bez zmniejszenia oglnoci rozumowania, e q > p, czyli q p + 1. Wtedy k =
3p1
q1

3q4
q1
<
3q3
q1
= 3.
Moe by wic k = 1 lub k = 2. Gdyby byo k = 1, wtedy 3p 1 = q 1 i q = 3p wbrew temu, e q jest pierwsze.
W takim razie k = 2, czyli 3p 1 = 2q 2. Wyliczywszy z tego p i podstawiwszy do uamka
3p1
q1
otrzymujemy po
drobnych przeksztaceniach, e 2p2[p+9, czyli p+9 2p2, a std ju mamy, e p 11 Dla p 5, 7 powysza
podzielno nie jest speniona, wic zostaje p = 11, skd wyliczmy, e q = 17. W prosty, cho do mudny sposb
dochodzimy do wniosku, e liczby te speniaj warunki zadania.
Przedstawione poniej zadanie (z IMO 2003) jest klasycznym przykadem odpowiedniego doboru rzdu:
Niech p bdzie liczb pierwsz. Pokaza, e istnieje taka liczba pierwsza q, e dla kadego n Z
+
zachodzi q n
p
p
Rozwizanie
Gdyby byo q = p, wtedy n = p nie speniaoby warunkw zadania, mamy wic (q, p) = 1. Wystarczy wic, e
rozpatrzymy takie n, e (n, q) = 1. Niech g bdzie generatorem mod q i g
a
n (mod q), g
b
p (mod q). Wtedy
kongruencja n
p
p (mod q) ma rozwizanie jeli ap b (mod q 1) ma rozwizanie (w a), co na pewno zachodzi
6
gdy p jest odwracalne mod q 1, musi wic by nieodwracalne, czyli (q 1, p) > 1 p[q 1, a jako e p jest
pierwsze, musi by p[q 1.
Jeliby wic teraz q[n
p
p, byoby p
q1
p
(n
p
)
q1
p
n
q1
1 (mod q) z MTF. Jeeli wic pokaemy, e istnieje
takie q, dla ktrego p[q 1 i ord
q
p
q1
p
, bdzie to dane q. Pokaemy nawet, e istnieje q speniajce te warunki
takie, e ord
q
p = p (wtedy drugi warunek zapisuje si jako p
2
q 1). Zauwamy, e ord
q
p = p wtedy i tylko wtedy,
gdy p
p
1 (mod q) (wtedy moliwe rzdy to p i 1) oraz p , 1 (mod q), czyli wtedy, gdy
p
p1
+ +p
2
+p + 1 0 (mod q).
Niech r bdzie dowolnym dzielnikiem pierwszym powyszej sumy. Udowodnilimy w pierwszym przykdzie tego
paragrafu, e zachodzi pierwszy zadany warunek, gdyby jednak p
2
[r 1, caa ta suma przystawaaby mod p
2
do 1,
co jest niemoliwe, gdy przystaje do p + 1, a p
2
p, a wic drugi warunek rwnie zachodzi.
Zadania
1. Znale wszystkie n Z
+
takie, e n
2
[2
n
+ 1
2. Znale wszystkie liczby pierwsze p, q takie, e pq[(5
p
2
p
)(5
q
2
q
)
3. Danych jest k liczb n
i
Z
+
(i 1, 2, . . . , k) parami wzgldnie pierwszych oraz liczba a Z
+
taka, e dla
kadego i 1, 2, . . . , k n
i
a 1 oraz n
i
[a
n
i
1. Pokaza, e istnieje co najmniej 3
k
1 takich liczb x Z
+
(x > 1), e x[a
x
1
4. Niech f(n) oznacza najwikszy wsplny dzielnik wszystkich liczb postaci a
n
a przy a Z i a > 1. Wyznaczy
wzr na f i pokaza, e jeli 2[n, to f(n) = 2
5. **Znale wszystkie n Z
+
, n > 1 takie, e istnieje dokadnie jedno cakowite a w przedziale [1, n!] takie, e
n![a
n
+ 1
6 Waluacje
Przedstawiony we Wstpie lemat o waluacjach jest pomocny przy dowodach podzielnoci skomplikowanych wyrae.
Zobaczmy to na prostym (na razie) przykadzie:
Dowie, e maksymalna potga w jakiej 3 dzieli n! jest rwna najwyszej potdze, w jakiej 3 dzieli = (1 + 4)(1 +
4 + 4
2
) . . . (1 + 4 + + 4
n1
).
Rozwizanie:
Oczywicie v
3
(n!) =

n
i=1
v
3
(i), za jako e 3 1 i 3 4, a 3[41, v
3
() =

n
i=1
(v
3
(4
i
1
i
)v
3
(41)) =

n
i=1
v
3
(i),
co koczy dowd.
Zazwyczaj jednak jest to tylko narzdzie pomocnicze przy dowodach tez bardziej wyranowanych. Jako przykad
udowodnimy nastpujce uoglnienie Zadania 1. z poprzedniego paragrafu
5
: Znale wszystkie x Z
+
, x > 1 i
p P takie, e x
p1
[(p 1)
x
+ 1
Rozwizanie
Dla p = 2 otrzymujemy x = 2, niech teraz p > 2. Niech q bdzie najmniejszym dzielnikiem pierwszym x. Mamy w
szczeglnoci (p 1)
x
1 (mod q), a std (p 1)
2x
1 (mod q). Niech t = ord
q
x. Z powyszych kongruencji
wynika, e t x i t[2x, za z wasnoci rzdu t[q 1. Jednake z wyboru q mamy (q 1, x) = 1, a wic z powyszych
faktw wynika, e t[2, czyli (p 1)
2
1 (mod q). To z kolei oznacza, e q = p lub p 2 (mod q). W drugim
przypadku mamy jednak, podstawiwszy go do tezy, q[2, czyli q = 2, jednake gdyby x byo parzyste, nieparzyste
musiaoby by p 1, sprzeczno. A wic p = q i std p[x.
Teraz (nareszcie) wchodz waluacje. Z zadanej podzielnoci wynika, e (p 1)v
p
(x) v
p
((p 1)
x
+ 1). A skoro
p p 1, p 1 i p[p 1 (1) = p, to z lematu o waluacjach (i wczeniej poczynionego spostrzeenia, e x musi
by nieparzyste), mamy:
v
p
((p 1)
x
+ 1) = v
p
((p 1)
x
(1)
x
) = v
p
(p 1 (1)) +v
p
(x) = v
p
(p) +v
p
(x) = v
p
(x) + 1.
Podstawiwszy to do otrzymanej nierwnoci, uzyskujemy (p 1)v
p
(x) v
p
(x) +1, a wic (p 2)v
p
(x) 1. Jako e
v
p
(x) 1, tym bardziej (p 2) 1, czyli p 3. Jako e x jest nieparzyste, musi by p = 3, za teza dla p = 3 to
Zadanie 1. z poprzedniego paragrafu.
Zadania
1. Niech n Z
+
i 2 n. Dowie, e zachodzi:
((n 1)
n
+ 1)
2
[n(n 1)
(n1)
n
+1
+n.
5
Jeli Czytelnik nie myla jeszcze nad tym wiczeniem, czytanie poniszego dowodu zepsuje Mu przyjemno rozwizywania tego
wcale nietrudnego problemu
7
2. Niech a Z, a > 3. Pokaza, e dla kadego m Z
+
istnieje takie n, e n[a
n
1 i n posiada dokadnie 2010
rnych dzielnikw pierwszych.
7 Teoria Grup i dalsze uoglnienia
Dalsze uoglnienia przydaj si rzadko. Czasem jednak zastosowanie Teorii Grup jest ciekaw alternatyw do
ocjalnego olimpijskiego rozwizania. Najpierw pokrtce, bez dowodw, objani podstawy Teorii:
1. Grup nazywamy par (zbir, dziaanie
6
) tak, e w zbiorze istnieje element neutralny (tj taki, ktry pomnoony
przez dowolny element zbioru daje ten element), kady element zbioru ma swoj odwrotno w zbiorze (tj.
taki element, ktry pomnoony przez dany daje element neutralny), oraz iloczyn dowolnych dwch elementw
zbioru naley do zbioru.
2. Rzdem grupy nazywamy jej rozmiar, przyczym dowolny element pomnoony przez siebie tyle razy, ile wynosi
rzd grupy daje element neutralny
3. Podgrup grupy G = (A, ) nazywamy kad grup G
t
= (A
t
, ) tak, e A
t
A
4. Zachodzi Twierdzenie Lagrangea: rzd podgrupy dzieli rzd grupy.
Zauwamy, e dotd operowalimy na grupie (Z

p
, ), twierdzenie Lagrangea nazywajc wasnoci rzdu.
Rozwizywanie zada olimpijskich metod Teorii Grup polega na zdeniowaniu pewnej operacji na elementach
pewnego typu, a nastpnie na redukcji przypadkw za pomoc Twierdzenia Lagrangea. Jak nietrudno si domyli,
jest to metoda rozwizywania zada kombinatorycznych. A oto i przykad:
Na konferencji matematykw kady z uczestnikw ma pewn ilo znajomych (jeli a zna b to b zna a). Na wsplny
obiad zastawiono dwa due stoy. Kady z matematykw domaga si, by spoywa posiek przy jednym stole z parzyst
liczb swoich znajomych. Na szczcie udao si rozmieci matematykw wedle ich yczenia
7
. Pokaza, e ilo
moliwych rozmieszcze matematykw jest cakowit potg dwjki.
Rozwizanie:
Usadmy matematykw wedug poprawnego rozmieszczenia. Niech nakaz bdzie cigiemn liczb a
i
takich, e a
i
= 0
jeli i-ty matematyk ma zosta przy swoim stole, za a
i
= 1 gdy ten ma przenie si do drugiego stou (n - liczba
matematykw). Mwimy, e wykonujemy iloczyn nakazw A i B (zapisujemy A B), jeli kaemy matematykom
najpierw wykona nakaz A, a potem B (oczywicie iloczyn nakazw jest przemienny). Niech ponadto I = (0, 0, . . . , 0)
oraz S bdzie zbiorem wszystkich nakazw takich, e wykonanie kadego z nich przeksztaca rozmieszczenie zgodne
z warunkami zadania w inne (niekoniecznie rne) rozmieszczenie rwnie zgodne z warunkami zadania. Pokaemy,
e (S, ) jest grup.
Elementem neutralnym jest I
Kady nakaz jest swoj wasn odwrotnoci
Jeli a
i
= 0, to istnieje parzycie wiele przyjaci i, ktrzy maj si przenie, analogicznie jeli a
i
= 1 to istnieje
parzycie wiele przyjaci i ktrzy maj zosta. Mnoenie nakazw polega na dodawaniu odpowiednich liczb
(mod 2). Rozwamy nakazy A i B. Niech F
i
oznacza zbir przyjaci i, ktry maj instrukcj przeciwn ni
i w A, analogicznie V
i
dla B i Ui dla A B. Teraz U
i
jest rnic symetryczn zbiorw F
i
i V
i
jeli a
i
= b
i
i
iloczynem w przeciwnym wypadku, a wic skoro moce tych zbiorw byy parzyste, [U
i
[ te jest parzysta.
Niech teraz H
1
= I, T
1
, gdzie T
1
S T
1
,= I, a T
2
SH
1
. (H
1
, ) jest grup. Rozpatrzmy zbir T
2
X : X H
1
.
Oba elementy H
1
byy rne, wic i oba elementy rozpatrywanego zbioru s rne (aby to sprawdzi, wystarczy oba
pomnoy przez T
1
2
). Ponadto, jeli A, B H
1
, to jeliby T
2
A = B, wtedy byoby T
2
= BA
1
H
1
, sprzeczno. A
wic H
1
i rozpatrywany zbir s rozczne, co wicej, iloczyn kadego z elementw H
1
z elementem rozpatrywanego
zbioru naley do tego ostatniego (z konstrukcji), a wic oba razem tworz czteroelementowy zbir H
2
taki, e (H
2
, )
jest grup. Idc dalej takim samym rozumowaniem, usyskiwa bdziemy coraz to wiksze podgrupy G takie, e
[H
i
[ = 2
i
. Std rzd G musi by potg dwjki, co naleao udowodni.
Powyszy przykad jest do abstrakcyjny: znacznie atwiej jest, gdy bierzemy pod uwag znane obiekty matematyczne,
o ktrych w miar atwo jest si przekona, e tworz grup. Tak metod zastosujemy w kolejnym przykadzie,
tym razem... z informatyki:
Majc dan na wejsiu liczb n Z
+
, 1 < n 10
18
wypisa okres cigu Fibonacciego wzitego mod n
Rozwizanie:
8
6
zwane dalej mnoeniem
7
okazuje si, e zawsze si da - dowd tego faktu nie angauje jednak Teorii Grup i nie jest przez to dla nas ciekawy
8
To rozwizanie powstao na kku informatycznym krakowskiego V LO prowadzonym przez dra Witolda Jarnickiego. Uwzgldnia
kilka znanych faktw, z ktrych dowd kadego jest elementarny.
8
Znanym faktem jest, e dla cigu zdeniowanego rekurencyjnie jako a
n+2
= a
n+1
+a
n
zachodzi:
_

1 0
_
n

_
a
2
a
1
_
=
_
a
n+2
a
n+1
_
oraz, co si z tym wie, na podstawie dowolnych dwch kolejnych elementw tego cigu da si odtworzy wszystkie
po nich nastpujce elementy.
Poniewa drugi czynnik w zapisanym powyej rwnaniu jest stay, wystarczy znale najmniejsze k takie, e pierwsza
z macierzy podniesiona do k-tej potgi da siebie sam (i odj od wyniku 1).
Macierze m m o niezerowym wyznaczniku wraz z mnoeniem stanowi grup. Oczywicie fakt ten, wzity mod
n, dalej zachodzi: to rwnie mona atwo sprawdzi, przeliczajc wyznacznik z denicji. Teraz dodatkowo grupa
ta jest skoczona, a wic zachodzi twierdzenie Lagrangea, a to, o co pytamy, to rzd macierzy
_
1 1
1 0
_
(cilej: rzd
podgrupy zoonej z kolejnych potg mod n tej macierzy - jak wspomniano, w kadej grupie zachodz te same
wasnoci co w grupach rozwaanych w poprzednich rozdziaach).
Majc sfaktoryzowany rzd grupy, korzystajc z twierdzenia Lagrangea bardzo atwo jest policzy rzd elementu:
wystarczy zmniejsza kolejno wykadniki potg kolejnych liczb pierwszych w faktoryzacji rzdu grupy i rcznie
sprawdza, czy element podniesiony do potgi o tak uzyskanym wykadniku daje element neutralny, jeeli ju nie
daje, naley zwikszy w wykadnik o 1 i zacz zmniejsza kolejny wykadnik. Zajmuje to czas liniowo zalenby
od sumy tych wykadnikw, czyli logarytmiczny od rzdu grupy.
Wrmy do wyjciowego problemu: to, co zostao do wyznaczenia, to rzd rozpatrywanej grupy, czyli ilo macierzy
o elementach cakowitych dodatnich mniejszych od n o niezerowym wyznaczniku mod n, tj. ilo takich czwrek
(a, b, c, d) 0, 1, . . . , n 1
4
e ab , cd (mod n).
Zastanwmy si, na ile sposobw mona wybra a, b przy ustalonej wartoci ab = x. Przy ustalonym a, b mona
wybra dokadnie na (a, n) sposb jeli q = (a, n)[(x, n) (naley wtedy wzi b xa
1
(mod
n
q
), co daje q liczb
mod n), za w ogle nie da si wybra w przeciwnym wypadku. W sumie wic ab mona wybra na
n1

a=0
(a, n) = n +

d]n
(
n
d
) d
sposobw. Rnych uporzdkowanych par (ab, cd) jest oczywicie n(n 1), a wic po pomnoeniu powyszej liczby
przez n(n1) otrzymamy szukany rzd grupy. Jest on wielomianowo zaleny od n, a wic majc go sfaktoryzowanego,
moemy policzy rzd elementu w czasie O(log n).
Uzyskany wynik ciko jest jednak szybko policzy: tu w sukurs przychodzi obserwcja, e jeli n =

i
i
, to okres
cigu mod n jest to najmniejsza wsplna wielokrotno okresw cigu mod p

i
i
. Rzeczywicie, majc dane wartoci
cigu mod kade p

i
i
, z Chiskiego Twierdzenia o Resztach jestemy w stanie odtworzy warto cigu mod n i
rzeczywicie, dopki wszystkie te wartoci nie osign zera, warto mod n te zera nie osiagnie (wynika to z dowodu
tego twierdzenia). Majc teraz n postaci p

, rzd grupy policzy i sfaktoryzowa jest atwiej. Wynosi on:


p

(p

1)(p

i=1
p
i1
(p 1) p
i
= p

(p

1)(p

+ (p 1)p
1
).
Ostatecznie, jestemy w stanie wykona zadanie w czasie potrzebnym na faktoryzacj n, co np. za pomoc heurystyki
RHO Pollarda mona uczyni w czasie O(
4

n).
Skoro mowa o zastosowaniu poruszanych w tej pracy kwestii w zadaniach algorytmicznych, nie sposb nie wspomnie
o zadaniu p.t. Istnienie N z AMPPZ 2008:
Dane s liczba pierwsza p < 10
9
oraz liczby 1 < m < 20 i 0 < a < p. Znajd takie n, dla ktrego n
n
+n
m
daje reszt
a modulo p (lub wypisz NIE, jeli taka liczba nie istnieje)
Rozwizanie:
Zamy, e dla pewnego g-generatora mod p ag
m
, 0 (mod p). Jeli takie g nie istnieje, oznacza to, e wielomian
x
m
a ma co najmniej tyle pierwiastkw mod p, ile jest generatorw mod p, czyli (p 1), a wic e m (p 1).
Dla m = 20 najwiksze p speniajce t nierwno wynosi ok. 200 i dla p tej wielkoci zadanie mona rozwiza
metodami brutalnymi.
Jeeli jednak takie g istnieje, podstawmy kp + g za n dla pewnego k. Niech teraz g
m
a g
e
(mod p). Chcemy,
aby (kp + g)
kp+g
g
e
(mod p), co po uproszczeniu daje k + g e (mod p 1), a std k e g (mod p 1).
Tak wyznaczone n (rwne kp + g), jak wida, spenia warunki zadania. Kroki, ktre naley tu wykona to po
pierwsze znalezienie generatora, po drugie znalezienie tzw. wskanika, czyli e takiego, eby g
e
przystawao mod p
do okrelonej wartoci.
1. Generatorw, na mocy jednego z lematw udowodnionych przy okazji dowodu istnienia generatorw, jest (p
1). Stosujc analiz propabilistyczn mona wykaza, e algorytm losujcy dowoln liczb z Z

p
i sprawdzajcy
jej rzd wspomnianym we wczeniejszym przykadzie algorytmem jest cakiem efektywny; praktyka pokazuje,
e dla p rzdu 10
9
wykonuje on rednio zaledwie kilkanacie sprawdze.
9
2. Wskaniki znajduje si za pomoc tzw. algorytmu baby step - giant step. Jego idea polega na trzymaniu
w pamici wartoci A = (g
0
, g
1
, . . . , g

p1
) oraz B = (g

p1
, g
2

p1
, . . . , g
p1
) Jeeli chcemy znale
wskanik liczby a, bierzemy dla kadego i [0,

p 1|] szukamy liczby a(g


i
)
1
w cigu B. Poniewa kad
liczb (wykadnik) z przedziau [0, p1] da si zapisa jako i+j

p 1|, przy ktrym z i nasze poszukiwanie


zakoczy si sukcesem. Poniewa uywamy wyszukiwania binarnego, pesymistyczna zoono tego algorytmu
wynosi O(

p log p).
Zadanie to angauje wikszo prostych algorytmw teorioliczbowych, ktre kady, kto ma styczno choby z
Olimpiad Informatyczn powinien umie zaimplementowa. Implementacja rozwizania tego zadania z komentarzami
znajduje si w Dodatku B do niniejszej pracy.
8 Zadania rne
1. Niech p bdzie liczb pierwsz i d[p1. Znale wszystkie k Z
+
, dla ktrych istnieje a takie, e p[(a+k)
d
a
d
2. n 2 jest liczb naturaln oraz p = 2
n
+ 1. Dowie, e jeli p[3
p1
2
+ 1, to p jest liczb pierwsz.
3. Znale wszystkie pary liczb pierwszych (p, q) takich, ze pq[2
p
+ 2
q
4. Wykaza, e jeli (2
m
1)
2
[2
n
1, to m(2
m
1)[n
5. Rozwiza rwnanie:
x
7
1
x 1
= y
5
1
w (x, y) cakowitych.
6. Niech n, d Z
+
i d bdzie najmniejsz tak liczb, e dla kadego a wzgldnie pierwszego z n mamy n[a
d
1.
Pokaza, e istnieje taka liczba b Z
n
, dla ktrej ord
n
b = d.
7. *Niech g bdzie generatorem Fibbonacciego mod p = 4k + 3 (p P), tj. takim generatorem, e g
2
g + 1
(mod p). Pokaza, e g 2 rwnie jest generatoem mod p.
8. Pokaza, e istnieje liczba naturalna n taka, e n[2
n
+ 1 i n ma dokadnie 2010 dzielnikw pierwszych.
9. *Niech n, m Z
+
, m > 1 i n[a
m
1 dla kadego a wzgldnie pierwszego z n. Dowie, e:
n 4m(2
m
1)
oraz zbada, kiedy zachodzi rwno.
9 O zbiorze generatorw
Majc za sob przepraw przez zadania olimpijskie, przyjrzyjmy si narzdziom, ktrymi si posugiwalimy: moemy
zapyta na przykad o wasnoci zbioru generatorw mod p, czy oglniej, zbioru generatorw dowolnej grupy
cyklicznej.
Zanim przystpimy do dziaania, miejmy na uwadze prosty, wielokrotnie ju sygnalizowany fakt: jeli podniesiemy
wszystkie generatory do potgi k wzgldnie pierwszej z p1, zmienimy tylko ich kolejno (g
k
dalej jest generatorem).
Jeli za podniesiemy wszystkie do potgi k bdcej dzielnikiem p 1, uzyskamy cykliczn grup reszt
p1
k
-
potgowych mod p, ktrej rzd bdzie wynosi po prostu (
p1
k
), a zatem sytuacj bardzo podobn do wyjciowej.
Przeto analizowa bedziemy zbir generatorw, nie za - ich potg. Oznaczmy go G.
Liczno [G[ = (p 1), gdy generator podniesiony do potgi wzgldnie pierwszej z p 1 te jest generatorem, a
takich potg jest (p 1) (por. Dodaetk A).
Iloczyn

gG
g 1 (mod p). Wynika to po prostu z faktu, e jeli g G, to g
1
rwnie.
Suma

gG
g (p 1) (mod p).
Dowd: Rozpatrzmy tzw. p1-szy wielomian cyklotomiczny
p1
(x) mod p, tj. taki wielomian unormowany, ktrego
jedynymi pierwiastkami s p 1-sze pierwiastki pierwotne z jednoci, co wzite nie w C, a w Z

p
oznacza, e jego
pierwiastkami s generatory mod p. Szukamy ich sumy, czyli wspczynnika przy [(p1)1]-szej potdze. Pokaemy
nawet wicej: e wspczynnik przy [n 1]-szej potdze
n
(x) zawsze jest rwny (n).
Wykorzystamy w tym celu fakt, e

d]n

d
(x) = x
n
1. Oto jego dowd: wielomiany po obu stronach s
unormowane, zatem wystarczy pokaza, e maj te same pierwiastki. Pierwiastki lewej strony to wszystkie pierwiastki
pierwotne stopnia d takiego, e d[n, a zatem ich n-te potgi s jedynkami (a aden si nie powtarza), z kolei
pierwiastki prawej strony to pierwiastki n-tego stopnia z jedynki, a zatem kady z nich jest pierwiastkiem pierwotnym
10
jakiego d-tego stopnia, gdzie d[n.
Niech teraz f(n) oznacza szukany wspczynnik. Jeli porwnamy wspczynniki obu wielomianw, dostaniemy, e

d]n
f(d) = 0 jeli n > 1 i f(1) = 1, co jest rwnowan denicja funkcji .
Sugerowane w powyszym rozwizaniu przejcie z Z

p
do C jest bardzo naturalne: formalnie mwic, grupa generatorw
Z

p
jest homomorczna z grup pierwiastkw p-tego stopnia z jednoci. Nieformalnie, pierwiastki pierwotne na okrgu
jednostkowym i generatory to dwa rne spojrzenia na ten sam problem.
Charakterystyk Dirichleta (ang. Dirichlet character) nazywamy dowoln funkcj : Z
p
C tak, e [(x)[ = 1 dla
x ,= 0, (0) = 0, (1) = 1 i (a)(b) = (ab). atwo zauway, e pojcia rzdu i generatora znajduj zastosowanie
rwnie w wiecie charakterystyk Dirichleta. atwo te zauway, e s one wygodniejsze w algebraicznych obliczeniach.
Niech
k
(x) = 1, gdy x jest reszt
p1
k
-potgow mod p i 0 w przeciwnym wypadku. Jak zostao wspomniane,
wystarczy bra k takie, e k[p 1;
k
(x) = 1 rwnoway teraz k[
p1
ord
p
x
. Niech teraz (x) = 1 gdy x jest generatorem
mod p i 0 w przeciwnym wypadku. Z przytoczonej wczeniej rwnowanej denicji funkcji i z powyszych obserwacji
mamy, e

k]p1
(k)
k
(x) = (x).
Z drugiej strony, jest dokadnie k reszt
p1
k
-potgowych, co w jzyku charakterystyk Dirichleta brzmi: istnieje
dokadnie k charakterystyk takich, e
k
=
0
, gdzie
0
to charakterystyka dla kadego argumentu przyjmujaca
1. Wynika std, e
k
(x) =
1
k

k
=
0
(x). czc uzyskane fakty, uzyskujemy wzr funkcji charakterystycznej G:
(x) =

k]p1
(k)
k

k
=
0
(x).
Jednym z jego zastosowa jest uzyskiwanie oszacowa na najmniejszy generator mod p. Obliczenia pokazuj, ze jest
on rzdu log p, cho najlepsze grne oszacowanie, jakie udao si wykaza do tej pory, to O(p
1
4
) (Burgess). Pokaemy
tutaj oszacowanie rzdu p
1
2
, najlepsze, jakie da si w miar sprawnie uzyska poznan tutaj algebr.
Udowodnimy najpierw kilka lematw o charakterystykach Dirichleta.
Oznaczmy: = e
2i
p
oraz () =

p
k=1
(k)
k
(suma Gaussa).
Lemat 1: ()(t) =

p
k=1
(k)
kt
Dowd: Teza jest rwnowana wyraeniu: ()(t)(t) =

p
k=1
(k)(t)
kt
. Korzystajc z multiplikatywnosci oraz
z faktu, e [(x)[ = 1, uzyskujemy () =

p
k=1
(kt)
kt
. Zbir Z
p
i kt : k Z
p
s jednakowe mod p (oczywicie
zakadamy t ,= 0). Zatem tak po prawej, jak i po lewej stronie mamy (), a to koczy dowd.
Lemat 2: [()[ = p
1
2
.
Dowd:
[()[
2
= ()() =
p

k=1
()(k)
k
lemat1
=
p

k=1
p

l=1
(l)
kl

k
=
p

l=1
(l)
p

k=1

k(l1)
=
= (1) p +
p

l=2
(l)
l1
p1

k=0

k(l1)
= p +
p

l=2
(l)
l1
(
l1
)
p
1

l1
1
= p.
Lemat 3. Niech A Z
p
. Wtedy [

x,yA
(x +y)[ p
1
2
[A[.
Dowd: Z lematu 1. mamy, e: ()

x,yA
(x+y) =

x,yA

p
k=1
(k)
k(x+y)
. Wemy po obu stronach moduy
i zastosujmy nierwnos Cauchyego-Schwarza. Otrzymamy:
p
1
2

x,yA
(x +y)

x,yA
p

k=1
(k)
k(x+y)

=
p

k=1
[(k)[

xA

kx

yA

ky

_
_
_
p

k=1

xA

kx

2
p

k=1

yA

ky

2
_
_
_
1
2
.
Jednake
p

k=1

xA

kx

2
=
p

k=1

xA

kx
_

xA

kx
_
=
p

k=1

x
1
,x
2
A

k(x
1
x
2
)
=

x
1
,x
2
A
p

k=1
(
x
1
x
2
)
k
=
=

x
1
=x
2
A
p

k=1
(
x
1
x
2
)
k
+

x
1
,=x
2
A
(
x
1
x
2
)
(
x
1
x
2
)
p
1

x
1
x
2
1
= p [A[.
Analogicznie postpujemy z drug sum i otrzymujemy tez.
Oznaczmy teraz min(G) 1 = x. Oczywicie dla liczb n

1, 2, . . . , x mamy (n) = 0. Przyjmijmy A = 1, 2, . . . ,


_
x
2

.
11
Mamy:
0 =

k]p1
(k)
k

k
=
0

x,yA
(x +y) =
_
x
2
_
2
+

k]p1,k>1
(k)
k

k
=
0

x,yA
(x +y).
Po wziciu moduw i zastosowaniu lematu 3. otrzymujemy:
_
x
2
_
2
(2
m
1)p
1
2
_
x
2
_
,
gdzie m jest liczb rnych dzielnikw pierwszych p 1. Moemy wic doj do wniosku, e:
min G < 2
m+1
p
1
2
.
Oczywicie oszacowanie przez 2
m+1
jest bardzo grube, gdyby szacowa to subtelniej, monaby byo uzyska wynik
O(m
c
p
1
2
) przy pewnej staej c, jednake dla duych p ten czynnik i tak traci na znaczeniu.
Monaby si zastanawia jeszcze nad rozmieszczeniem (rozkadem) G w Z

p
. Okazuje si, ze dla duych p przy
maym
]G]
]Z

p
]
rozkad ten mona przyblia rozkadem Poissona, jednake dowd tego faktu wykracza poza ramy
niniejszej pracy, jest ponadto obwarowany wieloma dodatkowymi zaoeniami. Szacowanie min G i wyznaczanie
wspomnianego rozkadu s zatem wanymi problemami otwartymi, ktre mona rozwaa poznawszy zawart w
niniejszej pracy teori. Jednake najwaniejszym (a na pewno najbardziej znanym) problemem dotyczcym G jest
hipoteza Artina, powizana ju raczej z teori sit i hipotez Riemanna. Sprowadza si ona do nastpujcego zadania:
Dana jest liczba a Z niebdca kwadratem liczby cakowitej ani 1. Czy istnieje nieskoczenie wiele liczb pierwszych
p takich, e a jest generatorem mod p?
Powodzenia!
12
10 Rozwizania
10.1 Prawie jak logarytmy
1. Odpowied: (x, y, z) = (0, 1, 2)
Dla y = 0 lewa strona jest podzielna przez 2, ale nie przez 8, nie moze wic by szecianem, sprzeczno.
Dla y = 1 z tego samego argumentu otrzymujemy, e 2[x, sprawdzenie mod 9 pokazuje, e 3[x.
Dla y 2 mamy 5
x
z
3
(mod 49), czyli 5
14x
z
42
z
(49)
1 (mod 49). Poniewa 5 jest generatorem
mod 49, musi by 42[14x, czyli 3[x
Ponadto 2 y, w przeciwnym bowiem wypadku lewa strona byaby parzysta i niepodzielna przez 4.
Mamy wic x = 3k, czyli 7
y
= (z 5
k
)(z
2
+5
k
z +5
2k
), a wic istniej takie < , + = y, e z 5
k
= 7

oraz z
2
+5
k
z +5
2k
= 7

. Po podniesieniu do kwadratu pierwszej rwnoci i odjciu jej od drugiej otrzymujemy


3 5
k
z = 7

7
2
, skd wniosek, e 7

[z, czyli z pierwszego rwnania 7

[5
k
, a wic = 0.
Teraz dla k = 0 otrzymujemy dane rozwizanie, za dla k ,= 0 z drugiego rwnania (po podstawieniu
z = 5
k
+ 1) mamy 7

1 (mod 5), co jest niemoliwe gdy = y jest nieparzyste.


2. Odpowied: 1, 2, X.
Niech f bdzie dowolnym nierozkadalnym dzielnikiem P w Z[X]. Zamy najpierw, e X f(X) (w sensie
wielomianowym). Std i z wasnoci wielomianw wnioskujemy, e od pewnego miejsca dla kadej liczby
pierwszej p zachodzi p f(p). Bdziemy teraz rozpatrywa takie wanie, dostatecznie due p
Znanym faktem jest, e dla dowolego wielomianu W Z[X] i a, b Z zachodzi a b[W(a) W(b). Biorc
teraz, dla dowolnej liczby naturalnej k za a liczb p +k(p), za za b liczb p, otrzymujemy dla wielomianu f,
e kf(p)[f(kf(p) +p) f(p), a std f(p)[f(kf(p) +p). Z Twierdzenia Dirichleta, w postpie arytmetycznym
f(p)k +p istnieje nieskoczenie wiele liczb pierwszych (wszak (p, f(p)) = 1 z zaozenia), niech q bdzie jedn
z nich.
Mamy teraz f(p)[f(q)[2
q
2 = 2
kf(p)+p
2 z treci zadania. Ponadto, z treci zadania mamy 2
p
2 (mod f(p)),
a std p 1 (mod ord
f(p)
2), uwzgldniwszy to w powyszej podzielnoci mamy, e f(p)[2
kf(1)+1
2.
Jeli teraz f(1) ,= 0, prawa strona powyszej podzielnoci jest niezerowa, a wic f jest na modu ograniczony
przez sta warto, dla dowolnie duych p. Poniewa jednak jedyny ograniczony wielomian to wielomian
stay, f jest stae. Jeliby natomiast f(1) = 0, wtedy z tw. Bezouta X 1[f(X), a std i z warunkw zadania
otrzymujemy, e dla dowolnego p musi zachodzi p 1[2
p
2, co nie zachodzi choby dla p = 5.
W takim razie zaoenie X f(X) byo bdne, jednake jedynym nierozkadalnym wielomianem takim,
e X[f(X) jest wielomian X
n
dla dowolnego n Z
+
. Zoywszy wszystkie otrzymane informacje mamy,
e P(X) = c X
n
. Podstawiwszy w warunku zadania p rwne 2 uzyskujemy, e n = 0 lub n = 1, za
podstawiwszy za p dowolny dzielnik pierwszy c na mocy przykadu 1 z paragrafu 5.1 otrzymujemy p = 1 lub
p = 2, a wic c = 1 lub c = 2, przy czym ten drugi przypadek tylko przy n = 0, wszystko z dokadnoci
do moduu. Uzyskujemy w kocu, e P 1, 2, X, pierwsze dwa oczywicie speniaj warunki zadania,
trzeci spenia je na mocy MTF.
10.2 Bijekcja z kapelusza
1. Przykad: x
i
= g
(p1)+i(1)
i
mod p dla g-generatora mod p
Niech x
i
g
a
i
(mod p). Aby przykdowa permutacja speniaa warunki zadania, potrzeba i wystarcza, aby
dla dowolnych 1 k < l p 1 byo

k
i=1
a
i
,

l
i=1
a
i
(mod p 1), czyli p 1

b
i=a
a
i
dla dowolnych
a < b mniejszych od p. Zauwamy, e a
i
= (1)
i
(i) spenia ten warunek, gdy wtedy suma ta wynosi po
prostu
ba
2
. Aby, znajc g, x
i
dao si uzyska za pomoc elementarnych operacji arytmetycznych, z MTF
pomy x
i
= g
(p1)+(1)
i
i
. Oczywicie jest to permutacja, gdy dla i ,= j mamy x
i
,= x
j
(dla i, j jednakowej
parzystoci jest to wasno generatora, dla i, j rnej parzystoci musiaoby by p 1[i +j, a z racji tego e
i +j jest nieparzyste p = 2, sprzeczno).
2. Niech g bdzie generatorem mod p. Zauwamy najpierw, e

p1
i=1
a
i
g

p2
i=0
(g
p1
2
)
1
1 (mod p) (g
nie moe by reszt kwadratow, musi wic by niereszt czyli z kryterium Eulera g
p1
2
1 (mod p)) - jest
to tzw. twierdzenie Wilsona. Teraz zaomy wbrew tezie, e a
1
b
1
, a
2
b
2
, . . . , a
p1
b
p1
= Z

p
.W takim razie
1
p1

i=1
a
i
b
i

p1

i=1
a
i
p1

i=1
b
i
(1) (1) 1 (mod p),
a wic p = 2, sprzeczno.
Co ciekawe, zadanie to mona uoglni na dowolne p Z
+
1, 2. Dowd tego jest ciekawy, jednake nie
angauje metod omawianych w tej pracy; znale go mona na stronie:
http://www.mathlinks.ro/viewtopic.php?t=307060
13
3. a) Poniewa (t 1, p 1) = 1, istnieje takie s e s(t 1) 1 (mod p 1), czyli x
1+s(t1)
1 (mod p)
dla kadego x, a wic rwnie dla pewnego x S. Pokaemy indukcyjnie, e dla kadego m x
1+m(t1)
S.
Dla m = 1 wystarczy zastosowa trzeci wasno S dla t liczb x, za aby przej z m do m+ 1 wystarczy j
zastoswa do liczby x
1+m(t1)
i t 1 liczb x.
Podstawiajc w uzyskanym twierdzeniu s za m otrzymujemy 1 S, a wic aby udowodni tez wystarczy
wzi trzeci wasno dla danych dwch liczb i t 2 jedynek.
b) Odpowied: 1 gdy k[p 1 i 0 w przeciwnym wypadku
Z punktu (a) wynika, e iloczyn mod p dowolnej iloci elementw S naley do S, w szczeglnoci wic jeli
x S, to x
p2
S, czyli jeli x S, to multiplikatywna odwrotno x mod p te naley do S.
Niech g bdzie generatorem mod p i s > 0 bdzie najmniejsz tak liczb, e g
s
S. Jeliby teraz dla pewnego
y takiego, e g
y
S byo y = s +r przy r < s, to jako e (g
s
)

S, a wic g
s
S, mamy g
y
g
s
g
r
(mod p), a wic g
r
S, czyli wobec r < s jest r = 0.
Widzimy wic, e S jest zbiorem potg liczby g
s
mod p. Poniewa, jak pokazalimy w (a), 1 S, musi by
s[p 1. Ju jasno wida, e k = [S[ =
p1
s
, czyli s =
p1
k
, musi by wic k[p 1, wtedy s jest wyznaczone
jednoznacznie, a wic S jest wyznaczone jednoznacznie.
4. Odpowied: dla wszystkich liczb pierwszych prcz 2 i 3
Zastanwmy si najpierw, jak zmieniaj si liczby na tablicy podczas wykonywania ruchw. Niech t
i
oznacza
pocztkow warto liczby, ktra zostaa wymazana w i-tym ruchu (oczywicie t
1
, t
2
, . . . , t
p1
= Z

p
).
Pokaemy, e po wykonaniu i-tego ruchu wszystkie liczby na tablicy s wiksze o s
i
= t
i
+2t
i1
+4t
i2
+ +
2
i1
t
1
. Dla i = 1 jest to prawd, gdy wymazano t
1
i jej warto dodano do wszystkich pozostaych liczb.
Zamy, e nasza teza zachodzi dla pewnego i 1. Wtedy przed wykonaniem i+1-szego ruchu wszystkie liczby
s wiksze o s
i
ods swych pocztkowych wartoci, w szczeglnoci liczba ktra wanie ma zosta zmazana
wynosiu t
i+1
+s
i
. Do wszystkich pozostaych liczb dodamy wic t
i+1
+s
i
, czyli w sumie liczby te bd wiksze
od swych pocztkowych wartoci o t
i+1
+ 2s
i
= s
i+1
, co chcielimy pokaza.
Std wniosek, e po wykonaniu wszystkich p 1 ruchw liczba po prawej wynosi 0 + s
p1
= s
p1
=

p2
i=0
2
i
t
p1i
. Zadanie sprowadza si wic do znalezienia takich p, e istnieje taka permutacja Z

p
, e liczba
napisana powyej wynosi 0 mod p. atwo sprawdzi, e dla 2 i 3 teza nie zachodzi, niech p bdzie liczb
pierwsz rn od 2 i 3 i niech g bdzie generatorem mod p, niech ponadto 2 g

(mod p) oraz t
i
g

p1i
(mod p). Wtedy s
p1
=

p2
i=0
g
i+
i
. Poniewa mamy dowolno w wyborze tej permutacji, sprbujmy
najprostszego rozwizania:
i
= i. Teraz
s
p1

p2

i=0
g
i(+1)

p2

i=1
(g
+1
)
i

(g
+1
)
p1
1
g
+1
1
0 (mod p)
z MTF. Moglimy tak zapisa, gdy dla p > 3 zawsze istnieje taki generator, dla ktrego +1 , 0 (mod p1)
(tzn. zawsze istnieje wicej ni jeden generator, skd wynika moliwo takiego doboru). Udowodnilimy zatem,
e permutacja okrelona przez t
i
g
p1i
spenia warunki zadania, a wic w szczeglnoci istnieje.
10.3 Jeszcze o magicznych bijekcjach
1. Potrzeba i wystarcza, aby P(x) c

p1
i=1
(x i) + 1 (mod p) dla dowolnego c Z

p
Ustalmy y i zsumujmy
kongruencje zadania zapisane dla wszystkich wartoci x Z

p
. Na mocy faktu udowodnionego na pocztku
paragrafu, mamy wtedy (a
p1
) P(y) (a
p1
) (mod p), gdy zbir y, 2y, . . . , (p 1)y = Z

p
(wzite mod
p), gdy kademu x Z

p
odpowiada w tym zbiorze liczba y xy
1
.Poniewa a
p1
, 0 (mod p) z denicji
wielomianu, mamy P(y) 1 (mod p). Stad wielomian P(x) 1 ma pierwiastek mod p w kadym punkcie
i Z

p
, czyli P(x) c

p1
i=1
(x i) +1 (mod p). W zerze natomiast wielomian ten moe przyjmowa dowoln
warto.
2. Z denicji
_
a
p
_
a
p1
2
(mod p). Kady skadnik sumy po lewej stronie jest wic wartoci mod p pewnego
wielomianu

p1
i=0
a
i
x
i
w punkcie k, przy czym a
p1
=
_
a
p
_
. A zatem suma ta przystaje do
_
a
p
_
mod p. Jest
ona na pewno na modu mniejsza od p z denicji symbolu Legendrea, wic wynosi albo (p 1)
_
a
p
_
, albo

_
a
p
_
. Jednake w pierwszym z tych przypadkw wszystkie skadniki tej sumy prcz jednego rwnego zeru
musiayby by rwne 1 lub (wszystkie) 1. Ale wtedy ax
2
+bx+c musiaoby przyjmowa warto 0 dokadnie
w jednym punkcie, tzn. mie pierwiastek podwjny mod p, co przeczy zaoeniu, e = b
2
4ac , 0 (mod p)
(z wasnoci funkcji kwadratowej).
10.4 Dont underestimate the power of the right order
1. Odpowied: n 1, 3
Dla n = 1 teza zachodzi. Zamy n > 1 i niech p bdzie najmniejszym dzielnikiem pierwszym n (oczywicie
14
p ,= 2), wtedy (n, p 1) = 1. Niech t = ord
p
2. Poniewa 2
2n
1 (mod p), z wasnoci rzdu mamy t[2n i
t[p 1, a wic t[(2n, p 1) = 2, czyli t = 1 (sprzeczno) lub t = 2 i wtedy p = 3.
Niech teraz q bdzie najmniejszym rnym od 3 dzielnikiem n, wtedy (n, q 1) 1, 3. Jak poprzednio
dochodzimy do wniosku, e ord
q
2 1, 2, 3, 6, czyli odpowiednio q 1, 3, 7 (w ostatnim przypadku mamy
wszak q[63). Std wniosek, e q[7, ale skoro n = 3k, to 2
n
+1 = 8
k
+1 1 +1 2 , 0 (mod 7), sprzeczno.
Mamy wic, e n = 3

.
Teraz liczymy na palcach, e 2 jest generatorem mod 3
2
, a wic z jednego z przytoczonych we Wstpie
lematw mamy, e 2 jest generatorem mod 3
2
. Ale skoro ma by 2
3

1 2
(3
2
)
2
(mod 3
2
), to musi by
(z wasnoci generatora) 3

(3
2
)
2
(mod (3
2
)), czyli po prostu 2 3
21
[2 3

, co jest oczywicie moliwe


tylko wtedy, gdy = 1, a wic gdy n = 3.
2. Odpowied: p, q 3, 3, 3, 13
Od razu widzimy, e adna z tych liczb nie moe by rwna 2 ani 5. Zauwamy te, e z MTF 5
p
2
p
52 3
(mod p), A wic jeli p ,= 3 to p[5
q
2
q
i analogicznie, jeli q ,= 3, to q[5
p
2
p
. Jeeli jedna z tych liczb jest
rwna 3, to, jak atwo sprawdzi, druga jest rwna 3 lub 13. Pokaemy, e jeli p, q ,= 3 to rozwiza nie ma.
Bez straty oglnoci rozumowania moemy przyj, e p q. Wiemy, e 2
q
5
q
(mod p), czyli 1
_
5
2
_
q
(mod p). Niech t = ord
p
_
5
2
_
. Z wasnoci rzdu t[q oraz t[p 1, a wic t = 1 lub t = q, bo q jest pierwsze.
Jeli t = 1, mielibymy 5 2 (mod p), czyli p = 3 wbrew zaoeniu. Jest wic t = q, skd wynika, e q[p 1,
a wic w szczeglnoci q p 1 < p, czyli sprzeczno z zaoeniem.
3. Wida, e n
i
,= 1, niech wic p
i
bdzie najmniejszym dzielnikiem pierwszym n
i
. Pokaemy, e p
i
[a 1.
Zamy przeciwnie, e t
i
= ord
p
i
a > 1, niech wic q
i
bdzie dowolnym dzielnikiem pierwszym t
i
. Wtedy z
wasnoci rzdu q
i
[p
i
1, a wic q
i
< p
i
, a poniewa p
i
[n
i
[a
n
i
1, q
i
[t
i
[n
i
z wasnoci rzdu. Czyli n
i
ma
mnejszy od p
i
dzielnik pierwszy, sprzeczno.
Z wykazanej wasnoci i MTF wynika, e p
i
[a
p
i
1. A wic dla kadego c
i
ze zbioru S
i
= 1, p
i
, n
i
(n
i
,= p
i
z zaoenia, e n
i
a 1) mamy c
i
[a
c
i
1. Rozpatrzmy m =

k
i=1
c
i
(z kadego S
i
bierzemy jeden dowolny
element). Musi by a
m
(a
c
i
)
m
c
i
1 (mod c
i
) Jednake wszystkie wybrane c
i
s parami wzgldnie pierwsze,
wic skoro kongruencja ta zachodzi modulo kade z nich, zachodzi te modulo ich iloczyn, czyli a
m
1
(mod m). Spostrzeenie, ze moliwych m (nie liczc jedynki) jest 3
k
1 koczy dowd.
4. Odpowied:
f(n) =

p1]n1,pP
p
Jeli p jest pierwsze i p 1[n 1 to z MTF p dzieli wszystkie liczby zadanej postaci. Pokaemy, e jest to
warunek konieczny na to, by p[[f(n). Jeli pewne q P i q
k
[[f(n) to q
k
[q
k
q, a wic k = 1. Ponadto q[g
n
g
gdzie g jest generatorem mod q, a wic q[g
n1
1, czyli q1[n1 z wasnoci rzdu, co chcielimy dowie.
Jeli teraz n jest parzyste i p 1[n 1, p 1 musi by nieparzyste, co zachodzi tylko dla p = 2 i to dowodzi,
e f(2n) = 2.
5. Odpowied: n P
n = 2 dziaa, zamy n > 2. Jeliby n byo parzyste, lewa strona miaaby dzielnik pierwszy postaci 4k + 3,
ktrego prawa, jako suma kwadratw, mie nie moe, sprzeczno
9
.
Zamy, e n jest nieparzyst liczb zoon. Niech q P i q[n, mamy n 2q, a wic q[
n!
q
= l. Wtedy
(l + 1)
n
1 = l

n1
i=0
(l + 1)
i
, a skoro

n1
i=0
(l + 1)
i


n1
i=0
1 0 (mod q), mamy n![(l + 1)
n
1. Std
(a(l +1))
n
1 (mod n!), ale skoro a ma by jedynym rozwizaniem mod n!, musi by a(l +1) a (mod n!),
czyli n![l, a wic n! l, sprzeczno.
Zamy teraz, e n jest liczb pierwsz nieparzyst. a = n! 1 jest dobrym rozwizaniem, zamy e istnieje
inne: b. Oczywicie a i b musz by wzgldnie pierwsze z n!. Rozpatrzmy liczb x ab
1
(mod n!). Z warunkw
zadania mamy x
n
1 (mod n!). Niech t = ord
n
x, wtedy t[n. Poniewa n jest pierwsze, t = n lub t = 1. W
pierwszym przypadku z wasnoci rzdu n[(n!), ale w rozwiniciu (ze wzoru) (n!) wystpuj tylko liczby
pierwsze mniejsze od n, sprzeczno. Std t = 1, czyli ab
1
1 (mod n!), a std wida e a b (mod n!),
czyli rzeczywicie a jest jedynym rozwizaniem mod n! zadanej podzielnoci.
10.5 Waluacje
1. Niech f
a,b
(x) = a
x
b
x
. Z lematu o waluacjach (dla liczb a
y
, b
y
i wykadnika x) wynika, e jeli p[f
a,b
(y) i x[y
(przy p a i p b) to v
p
(f
a,b
(x)) = v
p
_
x
y
_
+v
p
(f
a,b
(y)) (dla p P 2 ).
Pokaemy, e jeli 2 a b i x[f
a,b
(x), to zachodzi
f
a,b
(x) [
x f
a,b
(f
a,b
(x))
f
a,b
(x)
def
= .
9
Jest to znany fakt, ktrego dowd znajduje si np. w [1], warto go te przeprowadzi jako proste wiczenie
15
Niech p bdzie liczb pierwsz tak, e p[f
a,b
(x). Z warunku rnej parzystoci liczb a i b wynika, e p ,= 2.
Mamy wic v
p
() = v
p
(x) +v
p
_
f
a,b
(x)
x
_
+v
p
(f
a,b
(x)) v
p
(f
a,b
(x)) = v
p
(f
a,b
(x)), co chcielimy dowie.
Teraz wystarczy zastosowa powyszy fakt dla a = (n 1), b = 1, x = f
a,b
(1).
2. Pokaemy, e jeli n[a
n
1 to istnieje takie pierwsze q, e q[a
n
1 i (n, q) = 1. Zastosujemy indukcj
matematyczn ze wzgldu na liczb rnych dzielnikw pierwszych n.
Jeli liczba ta wynosi 0, oczywicie istnieje takie p e p[a 1. Jeli a jest nieparzyste, musimy pozby si
niewygodnego warunku, e gdzie uzyskamy p = 2. Dlatego wemy teraz za p dwjk i pokaemy, e istnieje q
takie, e (q, 2) = 1 i q[a
2
1 = (a 1)(a +1), a poniewa (a +1, a 1) = 2, aby takie q nie istniao musiaoby
by a1 = 2, sprzeczno z zaoeniem. Teraz zamy, e istnieje n
k
takie, e n
k
[a
n
k
1 i p
k
P2 takie, e
(n
k
, p
k
) = 1 i p
k
[a
n
k
1. Chcemy dowie, e istnieje p
k+1
speniajce analogiczne warunki dla n
k+1
= n
k
p
k
.
Zamy przeciwnie, ze takie p
k+1
nie istnieje, czyli liczba
k+1
= a
n
k
p
k
1 ma te same dzielniki pierwsze q
co
k
= a
n
k
1. Jednake z lematu o waluacjach mamy v
q
(
k+1
) = v
q
(
k
) + v
q
(p
k
), a skoro p
k
jest pierwsze,
to

k+1

k
= p
k
, co jest niemoliwe gdy ten iloraz jest sum p
k
potg a, z ktrych nie wszystkie wynosz 1.
Teraz wystarczy za szukan liczb n wzi iloczyn p
1
p
2
. . . p
2010
, i liczba ta na mocy wyej udowodnionego
twierdzenia spenia warunki zadania.
10.6 Zadania rne
1. Odpowied: k Z
+
Zamy, e p a. Podzielno zadania jest rwnowana kongruencji
_
1 +
k
a
_
d
1 (mod p).
Niech g bdzie generatorem mod p. atwo ju sprawdzi, e:
k
g
p1
d
1
jest jej rozwizaniem, a wic dowolne k spenia warunki zadania.
2. Mamy 3
2
n1
1 (mod p), a stad 3
2
n
1 (mod p). Niech t = ord
p
3. Z wasnoci rzdu mamy, e t 2
n1
i
t[2
n
, a wic t = 2
n
. Z drugiej strony t[(p), a wic p 1[(p), a std ju wprost wynika teza.
3. Odpowied: p, q 2, 2, 2, 3
Zamy najpierw, e p ,= 2 ,= q. Zapiszmy p = 2
k
u + 1 oraz q = 2
l
v + 1 przy nieparzystych u i v i bez
zmniejszenia oglnoci rozumowania zamy l k. Z warunkw zadania mamy w szczeglnoci 2
q
2
p
(mod p), co z MTF daje 2
q1
1 (mod p), a std 2
2(q1)
1 (mod p). Niech t = ord
p
2. Z wasnoci rzdu
mamy t q 1 i t[2(q 1), a to na mocy powzitych oznacze daje v
2
(t) = l + 1. Z wasnoci rzdu mamy
jednak, e t[p 1, a wic w szczeglnoci t[2
k
, a wic l + 1 = v
2
(t) k, sprzeczno z zaoeniem. A wic co
najmniej jedna z liczb p, q musi by rwna 2. Niech bez straty oglnoci p = 2. Wtedy, jeli q ,= 2, mamy z
MTF q[4 + 2
q
q[4 + 2 = 6 q = 3. Jeli za q = 2, to mamy 4[8, co rwnie jest prawd.
4. Pokaemy najpierw, e m[n. Niech (m, n) = d i m = dm
t
, wtedy (m
t
, n) = 1. Mamy z warunkw zadania
2
m

1[2
m
1[2
n
1, niech v = 2
m

1. Z zapisanych tu podzielnosci wynika, e ord


v
2[m
t
oraz ord
v
2[n, ale
wobec (m
t
, n) = 1 musi by ord
v
2 = 1, czyli v = 1, co oznacza e m
t
= 1, a wic m = d, czyli m[n.
Udowdnimy teraz, e 2
m
1[
n
m
. Niech w = 2
m
1. Z lematu o waluacjach mamy, e dla kadego dzielnika
pierwszego q liczby w zachodzi v
q
(2
n
1) = v
q
(w) + v
q
_
n
m
_
. Z warunkw zadania mamy, e 2v
q
(w)
v
q
(2
n
1) = v
q
(w) +v
q
_
n
m
_
, a std v
q
(w) v
q
_
n
m
_
, a zatem w[
n
m
.
5. Odpowied: brak rozwiza
Niech p bdzie dzielnikiem pierwszym wyraenia
x
7
1
x1
i niech k = ord
p
x. Poniewa w szczeglnoci p[x
7
1,
k[7, a wic k = 1 lub k = 7. Jeli k = 1, to x 1 (mod p), skd 0
x
7
1
x1
= x
6
+ ... + x + 1 7 (mod p),
a wic p = 7. Jeli za k = 7, to z MTF mamy, e 7[p 1, czyli p 1 (mod 7). Zamy teraz, e istnieje
rozwizanie (x, y) danego rwnania. Rozpatrzmy najpierw przypadek, gdy x , 1 (mod 7). Wtedy z MTF
x
7
1
x1

x1
x1
1 (mod 7), a wic i y
5
1 1 (mod 7). Mona na palcach wyliczy, e wtedy y 4 (mod 7).
Ale wtedy y 1 3 (mod 7), a skoro y 1[y
5
1, to prawa strona posiada dzielnik postaci 7k + 3, a ten z
kolei musi posiada dzielnik pierwszy niebdcy postaci 7k lub 7k +1 (gdyby nie posiada, sam byby postaci
7k lub 7k + 1). A lewa strona, jak wykazalimy, takich dzielnikw p mie nie moze. Jeeli za x 1 (mod 7),
to 7[
x
7
1
x1
, a wic y
5
1 (mod 7), i znw na palcach (lub korzystajc z rzdw) sprawdzamy, e wtedy y 1
(mod 7) i wobec tego, e y
4
+ ... + y + 1[y
5
1 i y
4
+ ... + y + 1 5 (mod 7), mamy, e prawa strona ma
dzielnik pierwszy niebdcy postaci 7k lub 7k +1, ktrego lewa strona mie nie moe. std wniosek, ze nie ma
par (x, y) speniajcych zadane rwnanie.
16
6. Udowodnimy najpierw, e jeli n = 2
k
, to ord
n
5 = 2
k2
. Z pierwszego z przytoczonych we wstpie lematw
mamy w szczeglnoci, e 2
k
[5
2
k2
1. Zatem ord
n
5[2
k2
, czyli ord
n
5 = 2
v
. Zamy, e v k 3 - oznacza
to w szczeglnoci, e ord
n
5[2
k3
, czyli 2
k
[5
k3
1. Pokaemy teraz indukcyjnie, e 5
2
1
= 1 + 2
+1
h przy
nieparzystym h. Dla = 1 teza oczywicie zachodzi, zamy, e zachodzi dla pewnego . Mamy zatem:
5
2

= (1 + 2
+1
h)
2
= 1 + 2
+2
h + 2
2+2
h
2
= 1 + 2
+2
(h + 2

h
2
), skd jasno wynika teza. Stosujc uzyskany
wynik dla = k 2 mamy 5
2
k3
= 1 + 2
k1
h wbrew zaoeniu, e 2
k
[5
k3
1.
Jeli teraz n = 2
k
, to wida, e d = 2
k2
. Niech n = 2
k

s
i=1
p

1
i
. Jeli n[a
d
1, to p

i
i
[a
d
1. Podstawmy
za a generator mod p

i
i
. Uzyskujemy, e (p

i
i
)[d. Z Chiskiego Twierdzenia o Resztach istnieje a bdce
generatorem mod p

i
i
dla kadego i, musi wic by l = lcm[(p

1
1
), (p

2
2
), . . . , (p

s
s
)][d. Wemy teraz takie r,
e generator g mod p

r
r
ma najwikszy rzd t mod 2. Mamy wtedy d = tl = ord
n
g, co chcielimy pokaza.
7. Jeeli g jest generatorem mod p i (k, p 1) = 1, to g
k
te jest generatorem mod p na mocy jednego z lematw
udowodnionych wraz z dowodem istnienia generatorw.
g (g 1)(g +1) g
2
(g 1) (mod p), a wic g 1 g
1
g
p2
(mod p) (z MTF). Jako e (p2, p1) = 1,
g
p2
te jest generatorem mod p, a wic jest nim g 1. W szczeglnoci, g 1 nie jest reszt kwadratow
mod p, czyli 1 (g 1)
p1
2
(g 1)
2k+1
(mod p). Teraz (g 1)
2k+3
(g 1)
2
g
2
+ 2g 1 g 2
(mod p). Poniewa (2k + 3, p 1) = 1, (g 1)
2k+3
jest generatorem mod p, a wic g 2 rwnie.
8. Pokaemy najpierw, e jeli p jest liczb pierwsz i p[2
p
+1, to p = 3. Mamy 2
p
1 (mod p), i std 2
2p
1
(mod p). Niech wic t = ord
p
2. Widzimy, e t[2p i t p, wic t[2 (std mamy tez) lub t = 2p, a wtedy z
wasnoci rzdu t[p 1 i std sprzeczno.
Pokaemy indukcyjnie ze wzgldu na liczb dzielnikw pierwszych n, e jeli n[2
n
+ 1 to istnieje q takie, e
q[2
n
+ 1 i q n. Jeli liczba ta wynosi 0, teza zachodzi (p
1
= 3). Jeli za zachodzi dla pewnego k, wystarczy
e udowodnimy, e zachodzi dla n
k+1
= n
k
p
k
. Jeeliby jednak liczby n
k
p
k
i 2
n
k
p
k
+ 1 miay ten sam zestaw
dzielnikw pierwszych, byoby p
k
= 3 - sprzeczno.
Teraz wystarczy wzi liczb n = p
1
p
2
. . . p
2010
aby otrzyma tez.
9. Rwno zachodzi dla m = 2, n = 24 oraz dla m = 4, n = 240
Zauwamy najpierw, e jeeli (n, 2) = 1, to n[2
m
1, a wic n 2
m
1 < 4m(2
m
1) i zachodzi nierwno
ostra. Niech wic 2
x
[[n. Niech p bdzie liczb pierwsz nieparzyst i p

[[n i niech A = p P 2 : p[n.


Oczywicie wrd liczb a speniajcych warunki zadania jest generator g mod p

. Wobec g
m
1 (mod p

)
mamy, e (p

) = p
1
(p 1)[m. Wynika z tego, e p 1[m, a wic z MTF p[2
m
1. Poniewa wszystkie
liczby p A s wzgldnie pierwsze, moemy zapisa, e

pA
p[2
m
1. Z drugiej strony, mamy e p
1
[m,
analogicznie mnoc otrzymujemy, e
n
2
x

pA
p
[m. Uzyskujemy wic
n
2
x
[m(2
m
1). Poza tym, w zadaniu 6 z
tego rozdziau udowodnilimy, e ord
2
x5 = 2
x2
. Zatem podstawiwszy za a pitk, otrzymujemy 2
x2
[m, co
moemy, wobec nieparzystoci
n
2
x
, domnoy do naszej podzielnoci, dostajc n[4m(2
m
1), a std dan
nierwno.
Zamy teraz, e n = 4m(2
m
1). W takim razie nie tylko

pA
p[2
m
1, ale

pA
p = 2
m
1. Pokaemy, e m
nie moe mie dzielnika pierwszego nieparzystego. Istotnie, niech q bdzie takim dzielnikiem. Z zaoenia mamy
wic, e rwnie q[n, a std q A i, jak udowodnilimy wczeniej, q1[m. Mamy przeto, e (q
2
) = q(q1)[m,
a wic 2
m
= (2

)
(q
2
)
, a skoro 2 q to z twierdzenia Eulera 2
m
1 (mod q
2
), czyli q
2
[2
m
1, a zaoenie
mwi, e wszystkie dzielniki pierwsze 2
m
1 wystpuj w rozwiniciu tej liczby w pierwszej potdze. Wnosimy
std, e m = 2
l
. atwo dowie (indukcyjnie), e 2
2
l
1 =

l1
i=0
2
2
i
+ 1. A z przykadu z praragrafu 5.2 mamy,
e jeli l 3, to 2
2
l1
+ 1 ma dzielnik pierwszy postaci 2
l
s + 1. Std wnosimy, e 2
l
s + 1 A. A wic, jako
e dla kadego p A p 1[m, mamy 2
l
s[2
l
, skd wynika, e dla pewnego p A p 1 = m. Takie p moe
by oczywicie tylko jedno, wic 2
2
l1
+ 1 musi mie tylko jeden dzielnik pierwszy, czyli musi by pierwsze i
spenia t rwno, a wic 2
2
l1
= m = 2
l
. Std otrzymujemy l = 2. Nie wykluczylimy jeszcze l = 1. Dla
l 1, 2 rozwizania wynosz odpowiednio (2, 24) i (4, 240). atwo sprawdzi, e oba dziaaj.
17
11 Dodatki
11.1 Dodatek A - dowd istnienia generatorw
Twierdzenie: Liczba n ma generator wtedy i tylko wtedy, gdy jest postaci 2, 4, p

, 2p

, gdzie Z a p P \ {2}
Dowd:
Pokaemy najpierw, e adna inna liczba generatora mie nie moe. Niech n posiada generator g i niech p

1
1
p

2
2
...p

k
k
bdzie rozkadem
n na czynniki pierwsze. Niech N bdzie liczb podzieln przez (p

i
i
) dla kadego i. Dla kadego i z tw. Eulera mamy, e p

i
i
|g
(p

i
i
)
1,
a wic tym bardziej p

i
i
|a
N
1. Gdyby co najmniej dwie liczby (p

i
i
) byy parzyste, N mogoby by rwne
1
2
(p

1
1
)(p

2
2
)...(p

k
k
)
(bo wtedy przy dzieleniu jednego z parzystych czynnikw (p

i
i
) przez N potrzebna dwjka w rozwiniciu N znalazaby si przy innym
parzystym czynniku tej postaci). Ale wtedy (wobec multiplikatywnoci funkcji Eulera dla argumentw wzgldnie pierwszych) N =
1
2
(n),
a wic g
1
2
(n)
1 (mod n) wbrew zaoeniu, e g jest generatorem. Zauwamy jednak, e jeli p > 2, to 2|(p

) = p
1
(p 1), bo
2|p 1, a jeli p = 2 i > 1 to te 2|(p

) = 2
1
. Wida wic, e n musi by postaci p

, 2p

lub 2

, w przeciwnym bowiem razie


dwie liczby (p

i
i
) byyby parzyste.
Udowodnimy teraz:
Lemat 1: Dla kadej liczby nieparzystej a i cakowitej przy 3 mamy 2

|a
2
2
1.
Dla = 3 mamy 8|a
2
1 i atwo sprawdzi, e dla a nieparzystego tak wanie jest. Zamy teraz, e 2
1
|a
2
3
1 dla pewnego
3. Pokaemy, e w takim razie 2

|a
2
2
1 = (a
2
3
1)(a
2
3
+1). Pierwszy czynnik po lewej stronie jest z zaoenia podzielny
przez 2
1
, natomiast drugi wobec nieparzystoci a jest parzysty, wic cao jest podzielna przez 2

, co chcielimy pokaza.
Jeeliby teraz g byo generatorem mod 2

, musiaoby by ord
2
g = (2

) = 2
1
, co na mocy powyszego lematu jest dla 3
niemoliwe. Pokazalimy wic, e generator maj co najwyej liczby 2, 4, p

, 2p

.
atwo sprawdzi, e liczby 2 i 4 posiadaj generator (dla 2 jest to 1, dla 4 - 3).
Pokaemy najpierw, e kada liczba pierwsza p posiada generator.
Niech (t) oznacza liczb liczb naturalnych a mniejszych od p takich, e ord
p
a = t. Poniewa kada z p 1 liczb a posiada dokadnie
jeden rzd mod p, ktry (z wasnoci rzdu) jest dzielnikiem (p) = p 1, to:

t|p1
(t) = p 1. (1)
Dowiedziemy teraz:
Lemat 2:

d|n
(d) = n
Rozpatrzmy cig uamkw:
1
n
,
2
n
, . . . ,
n
n
, ale zapisanych w postaci nieskracalnej. Mianownik kadego z tych uamkw jest dzielnikiem
d liczby n, a licznik jest wzgldnie pierwszy z d. Uamkw o mianowniku d nie moe by w takim razie wicej ni (d). Nie moe by
ich te mniej ni (d), gdy jeli tylko dla pewnego x jest (x, d) = 1 i x d, to 1
n
d
x n, a wic uamek
x
d
=
n
d
x
n
jest elementem
rozpatrywanego cigu. A poniewa elementw cigu jest wci n, to istotnie

d|n
(d) = n, co koczy dowd lematu.
Stosujc Lemat 2. dla n = p 1 otrzymujemy wobec (1):

t|p1
(t) (t) = 0. (2)
Dowiedziemy, e dla kadego t jest (t) (t).
Jeli (t) = 0, to nierwno zachodzi. Zamy wic, e (t) = 0, a wic istnieje takie a, e ord
p
a = t.
Udowodnimy teraz:
Lemat 3: ord
p
a
k
= ord
p
a (ord
p
a, k) = 1.
Niech ord
p
a = t. Zamy najpierw, e (t, k) = 1 i niech ord
p
a
k
= s. Wtedy a
ks
1 (mod p) i z wasnoci rzdu t|ks, a wic na mocy
zaoenia t|s. Ale poniewa (a
k
)
t
(a
t
)
k
1 (mod p), to z wasnoci rzdu s|t i ostatecznie mamy s = t, co chcielimy otrzyma.
Zamy teraz, e (t, k) = d > 1 przy ord
p
a
k
= t. Wtedy t = t
1
d i k = k
1
d. A wic (a
k
)
t
1
a
k
1
dt
1
a
k
1
t
(a
t
)
k
1
1
k
1
1 (mod p).
Z wasnoci rzdu mamy, e t
1
|t, co wobec denicji liczby t
1
jest niemoliwe. Otrzymana sprzeczno koczy dowd lematu.
Jeli teraz dowiedziemy, e jeeli dla pewnego b ord
p
b = t, to b a
k
dla pewnego k, to bdziemy mieli, e takich liczb b musi by nie
wicej ni (k), bo tylko tyle k wobec lematu 3. wchodzi w rachub (dwie rne liczby b nie mog przecie przystawa do tego samego
a
k
), a wic bdziemy mieli dan nierwno. Wybrane przez nas b jest pierwiastkiem kongruencji x
t
1 (mod p). Kongruencja ta,
jako e jest stopnia t p 1, moe posiada co najwyej t rnych pierwiastkw. Poniewa ord
p
a = t, to liczby a
0
, a
1
, ..., a
t1
s
wszystkie rne i speniaj rzeczon kongruencj (bo spenia j samo a). Tak wic b musi przystawa do jednej z tych liczb, co chcielimy
pokaza.
Z otrzymanej nierwnoci wynika, e kady skadnik sumy po lewej stronie (2) jest nieujemny, a suma skadnikw nieujemnych moe
by rwna zeru wtedy i tylko wtedy gdy kady z nich jest rwny zeru. Mamy wic (t) = (t) dla kadego dzielnika t liczby p 1.
Podstawiajc za t samo p1, otrzymujemy, e kada liczba pierwsza p ma dokadnie (p1) generatorw mniejszych od p. Czyli liczby
pierwsze posiadaj generator.
Teraz pokaemy, e liczby postaci p

maj generator. Najpierw dowiedziemy prosty


Lemat 4: Niech g, n, s bd liczbami naturalnymi i g
n
= 1+kp
s
dla pewnego k Z
+
. Mamy wtedy: g
np
= 1+lp
s+1
dla pewnego l Z
+
i jeli p k to p l.
Poniewa g
np
= (1 + kp
s
)
p
= 1 +

p
1

kp
s
+

p
2

k
2
p
2s
+ ... i wszystkie skadniki tej sumy od trzeciego s podzielne przez p
s+2
, moemy
zapisa, e g
np
= 1+kp
s+1
+rp
s+2
= 1+p
s+1
(k+pr). Wida przy tym, e jeli p k, to p (k+pr), a wic lemat jest udowodniony.
Pokaemy, e jeli g jest generatorem mod p takim, e p
2
g
p1
1, to g jest generatorem mod p

.
Mamy, e g
p1
= 1 + kp i p k, a wic w myl lematu 4 g
p(p1)
= 1 + k
1
p
2
i p k
1
. Stosujc ten sam lemat jeszcze 3-krotnie
otrzymujemy g
p
2
(p1)
= 1 + k
2
p
1
i p k
2
, czyli g
p
2
(p1)
1 (mod p

). Niech t = ord
p
g. Z wasnoci rzdu mamy
t|(p

) = p p
2
(p 1). Ale t p
2
(p 1), wic musi by g = (), co chcielimy wykaza.
Pozostaje jeszcze udowodni, e dla kadego p istnieje generator mod p speniajcy zaoenia. Niech wic g
0
bdzie generatorem mod
p. Mamy wic g
p1
0
= 1 + kp. Rozpatrzmy kongruencj g
p2
0
x k 1 (mod p). Oczywicie ma ona rozwizanie, powiedzmy, x
0
. W
takim razie g
p2
0
x
0
= tp + k 1. Kada liczba g postaci g
0
+ x
0
p przystaje mod p do g
0
, wic jest generatorem mod p. Ale g
p1
=
18
(g
0
+x
0
p)
p1
= g
p1
+

p1
1

g
p2
0
x
0
p+

p1
2

g
p3
0
x
2
p
2
+... = g
p1
0
+(p1)(k1+tp)p+qp
2
= 1+p+(k1+tpt+q)p
2
= 1+p+lp
2
,
a wic g
p1
1 = p +lp
2
, a std ju mamy e p
2
g
p1
1, czyli poszukiwany generator istnieje, a wic liczby postaci p

rzeczywicie
posiadaj generator
Pokaemy na koniec, e kady nieparzysty generator g mod p

jest generatorem mod 2p

, i w konsekwencji e liczby postaci 2p

posiadaj generator.
Niech g
0
bdzie generatorem mod p

. Liczba g
0
+ p

przystaje do g
0
mod p

, a wic rwnie jest generatorem mod p

. Poniewa p

jest nieparzyste, to jedna z liczb g


0
i g
0
+ p

jest nieparzysta, a wic g speniajce zaoenia zawsze istnieje.


Niech ponadto ord
2p
g = t. Z wasnoci rzdu t|(2p

) = (2)(p

) = (p

), a wic w szczeglnoci t (p

). Z drugiej strony
mamy, e g
t
1 (mod 2p

), a wic tym bardziej g


t
1 (mod p

), wic (p

= ord
p
g|t z wasnoci rzdu. Std jasno wida, e g jest
generatorem mod 2p

, co chcielimy otrzyma.
Niniejszym dowd Twierdzenia zosta zakoczony.
11.2 Dodatek B - implementacja zadania istnienie N
Dane s liczba pierwsza p < 10
9
oraz liczby 1 < m < 20 i 0 < a < p. Znajd takie n, dla ktrego n
n
+ n
m
daje reszt a modulo p (lub
wypisz NIE, jeli taka liczba nie istnieje)
Implementacja rozwizania w C++:
#include <cstdio>
#include <ctime>
#include<algorithm>
using namespace std;
int random(int p) //losowanie duych liczb mod p
{
const int D=1000;
int d=1;
int wynik=1;
int q=p;
while(q>0)
{
int t=rand();
wynik+=(t%D)*d;
d=d*D;
q=q/D;
if(t%6==0)
break;
}
if(wynik%p==0) wynik=2;
if(wynik<0) wynik=(-1)*wynik;
return wynik%p;
}
int power(int n, int k, int p) //szybkie potgowanie n^k mod p
{
long long wynik;
if(k==0) return 1;
if(k==1) return n;
if(k%2==0)
{
wynik=(long long)power(n,k/2,p);
wynik=(wynik*wynik)%p;
}
if(k%2==1)
{
wynik=(long long)power(n,k-1,p);
wynik=(wynik*n)%p;
}
return (int)wynik
}
int reverse(int a,int p) //szukanie odwrtonoci a mod p
{
return power(a,p-2,p); //skoro a*a^(p-2)=a^(p-1)=1, to a^(p-2)=a^-1
}
int F[10000]; //tablica liczb pierwszych w rozkadzie
int w=1;
void Factorize(int p)
{
//rozkad p na czynniki pierwsze w O(sqrt(p)),
//szybciej nie ma sensu gdy i tak bs-gs zajmuje czas O(sqrt(p))
int q=p;
for(int i=2;i*i<=q;i++)
{
//sprawdzanie wszystkich liczb a do pierwiastka z p
if(p%i==0)
{
F[w]=i;
while(p%i==0) p=p/i;
if(p==1) return;
w++;
}
}
F[w]=p;
}
int order(int n,int p) // szukanie rzdu n mod p
{
int q=p-1;
if(n==1) return 1;
for(int i=1;i<=w;i++)
{
while(q%F[i]==0 && power(n,q,p)==1) q=q/F[i];
//zmniejszamy wykadnik kadej liczby pierwszej w rozkadzie p-1
if(power(n,q,p)!=1) q=q*F[i];
//a n podniesione do uzyskanego wykadnika przestanie dawa 1 mod p
}
return q;
}
int primitive_root(int p) // szukanie generatora mod p
{
//ustawienie bazy losowania na "losowy" sposb
srand((int)time(NULL));
int g=random(p);
// dopki nie znajdziesz generatora - losuj liczby z Zp*
while(order(g,p)!=p-1)
g=random(p);
return g;
}
struct point {int x;int y;};
bool operator<(point a, point b) {return(a.x<b.x);}
point A[100000];
point B[100000];
int BS(int s,int beg, int end) //wyszukiwanie binarne
{
int mid=(end+beg)/2;
if(A[mid].x==s) return mid;
if(beg>=end) return -1;
if(A[mid].x>s) return BS(s,beg,mid-1);
return BS(s,mid+1,end);
}
int bs_gs(int g,int p,int a) //baby step - giant step
{
if(a==1) return p-1;
int r=1; for(int i=1;i*i<p-1;i++) r++; //r=sqrt(p)
//konstrukcja cigw A i B
A[0].x=1; A[0].y=0; A[1].x=g; A[1].y=1;
long long t=(long long)(power(g,r,p))*(long long)g; t=t%p;
B[0].y=0; B[0].x=1; B[1].x=(int)t; B[1].y=1;
for(int i=2;i*i<p-1;i++)
{
long long a=(long long)A[i-1].x*(long long)A[1].x; t=t%p;
A[i].x=(int)a; A[i].y=i;
long long b=(long long)B[i-1].x*(long long)B[1].x; t=t%p;
B[i].x=(int)b; B[i].y=i;
}
19
//sortowanie A - w tej tablicy bdziemy wyszukiwa binarnie
sort(A,A+r+1);
for(int i=0;i<=r;i++)
{
long long t=(long long)reverse(B[i].x,p)*(long long)a;
t=t%p; t=(int)t;
int z=BS(s,0,r); //szukamy w A liczby a*(g^i)-1
if(z!=-1) return((r+1)*B[i].y+A[z].y);
//jeli poszukiwanie zakoczyo si powodzeniem,
//wypisujemy odpowied (a=g^i*g^z)
}
}
long long Brute_solve(int m, int a, int p)
{
//rozpatrywanie przypadkw maych p
int period=p*(p-1);
// poniewa szukamy k mod p-1 i r mod p (n=kp+r),
// z CRT wiemy, e wystarczy sprawdzi mod p(p-1)
for(int n=1;n<=period;n++)
{
int k=n/p;
int r=n%p;
int t=(a-power(r,m,p)+p)%p;
if(power(r,k+r,p)==t)
return (long long)n;
}
return (long long)-1;
}
long long Existing_solve(int m, int a, int p)
{
//Szukanie gdy wiemy, e istnieje g takie, e g^m-a!=0
Factorize(p-1);
int g=primitive_root(p);
while((a-power(g,m,p)+p)%p==0)
g=primitive_root(p);
long long l=(a-power(g,m,p))%p; while(l<0) l+=p; l=(int)l;
int e=bs_gs(g,p,l);
int k=(e-g)%(p-1); while(k<0) k+=(p-1);
return (long long)k*(long long)p+(long long)g;
}
long long solve(int m,int a,int p)
{
if(p<=211) return Brute_solve(m,a,p);
else return Existing_solve(m,a,p);
}
int main() //wczytanie danych
{
int z;
int s;
s=scanf("%d", &z);
for(int q=1;q<=z;q++)
{
int p,a,m;
s=scanf("%d %d %d", &p,&a,&m);
long long t=solve(m,a,p);
if(t==-1)
printf("NIE\n");
else
printf("TAK %lld\n", t);
}
return 0;
}
20
Literatura
[1] Wacaw Sierpiski, Teoria Liczb, Warszawa-Wrocaw 1950
[2] red. Tomasz Kobos, Jakub Konieczny i in., Archiwum Ligi Matematycznej V LO
[3] Titu Andreescu, Gabriel Dospinescu, Problems from the book, www.mathlinks.ro
[4] Problems in Elementary Number Theory, projectpen.wordpress.com
[5] Archiwum Olimpiady Matematycznej, www.om.edu.pl
[6] C. Cobeli, A. Zaharescu, On distribution of primitive roots mod p, Bukareszt 1998
[7] P. Erdos, N. Shapiro, On the least primitive root of a prime, Nowy Jork 1956
21

You might also like